GS Prelims Paper II General Comprehension up to 2019


Civil Services Examination 2019


Read the following seven passages and answer the items that follow each passage. Your answers to these items should be based on the passages only.

Passage - 1

Political theorists no doubt have to take history of injustice, for example, untouchability, seriously. The concept of historical injustice takes note of a variety of historical wrongs that continue into the present in some form or the other and tend to resist repair. Two reasons might account for resistance to repair. One, not only are the roots of injustice buried deep in history, injustice itself constitutes economic structures of exploitation, ideologies of discrimination and modes of representation. Two, the category of historical injustice generally extends across a number of wrongs such as economic deprivation, social discrimination and lack of recognition. This category is complex, not only because of the overlap between a number of wrongs, but because one or the other wrong, generally discrimination, tends to acquire partial autonomy from others. This is borne out by the history of repair in India.

Q. What is the main idea that we can infer from the passage?

  1. Untouchability in India has not been taken seriously by political theorists.
  2. Historical injustice is inevitable in any society and is always beyond repair.
  3. Social discrimination and deprivation have their roots in bad economies.
  4. It is difficult, if not impossible, to repair every manifestation of historical injustice.

Q. On the basis of the above passage, the following assumptions have been made:

  1. Removal of economic discrimination leads to removal of social discrimination.
  2. Democratic polity is the best way to repair historical wrongs.

Which of the above assumptions is/are valid?

  1. 1 only
  2. 2 only
  3. Both 1 and 2
  4. Neither 1 nor 2

Passage – 2

Education plays a great transformatory role in life, particularly so in this rapidly changing and globalizing world. Universities are the custodians of the intellectual capital and promoters of culture and specialized knowledge. Culture is an activity of thought, and receptiveness to beauty and human feelings. A merely well informed man is only a bore on God's earth. What we should aim at is producing men who possess both culture and expert knowledge. Their expert knowledge will give them a firm ground to start from and their culture will lead them as deep as philosophy and as high as art. Together it will impart meaning to human existence.

Q. On the basis of the above passage, the following assumptions have been made:

  1. A society without well educated people cannot be transformed into a modern society.
  2. Without acquiring culture, a person's education is not complete.

Which of the above assumptions is/are valid?

  1. 1 only
  2. 2 only
  3. Both 1 and 2
  4. Neither 1 nor 2

Passage – 3

Soil, in which nearly all our food grows, is a living resource that takes years to form. Yet it can vanish in minutes. Each year 75 billion tonnes of fertile soil is lost to erosion. That is alarming — and not just for food producers. Soil can trap huge quantities of carbon dioxide in the form of organic carbon and prevent it from escaping into the atmosphere.

Q. On the basis of the above passage, the following assumptions have been made:

  1. Large scale soil erosion is a major reason for widespread food insecurity in the world.
  2. Soil erosion is mainly anthropogenic.
  3. Sustainable management of soils helps in combating climate change.

Which of the above assumptions is/are valid?

  1. 1 and 2 only
  2. 3 only
  3. 2 and 3 only
  4. 1, 2 and 3

Passage – 4

Inequality is visible, even statistically measurable in many instances, but the economic power that drives it is invisible and not measurable... Like the force of gravity, power is the organising principle of inequality, be it of income, or wealth, gender, race, religion and region. Its effects are seen in a pervasive manner in all spheres, but the ways in which economic power pulls and tilts visible economic variables remain invisibly obscure.

Q. On the basis of the above passage, the following assumptions have been made:

  1. Economic power is the only reason for the existence of inequality in a society.
  2. Inequality of different kinds, income, wealth, etc, reinforces power.
  3. Economic power can be analysed more through its effects than by direct empirical methods.

Which of thy above assumptions is/are valid?

  1. 1 and 2 only
  2. 3 only
  3. 1 and 3 only
  4. 1, 2 and 3

Passage – 5

Climate change may actually benefit some plants by lengthening growing seasons and increasing carbon dioxide. Yet other effects of a warmer world, such as more pests, droughts, and flooding, will be less benign. How will the world adapt? Researchers project that by 2050, suitable croplands for four commodities — maize, potatoes, rice and wheat — will shift, in some cases pushing farmers to plant new crops. Some farmlands may benefit from warming, but others won't. Climate alone does not dictate yields; political shifts, global "demand, and agricultural practices will influence how farms fare in the future.

Q. Which one of the following is the most logical and rational inference that can be made from) the above passage?

  1. Farmers who modernize their methods and diversify their fields will be in an advantageous position in future.
  2. Climate change will adversely affect the crop diversity.
  3. Shifting major crops to new croplands will lead to a great increase in the total area under cultivation and thus an increase in overall agricultural production.
  4. Climate change is the most important factor affecting the agricultural economy in the future.

Passage – 6

A bat's wings may look like sheets of skin. But underneath, a bat has the same five fingers as an orangutan or a human, as well as a wrist connected to the same cluster of wrist bones connected to the same long bones of the arm. What can be more curious than that the hand of a man formed for grasping, that of a mole for digging, the leg of the horse, the-paddle of the porpoise, and the wing of the bat, should all be constructed on the same pattern?

Q. Which one of the following is the most logical, scientific and rational inference that can be made from the above passage?

  1. Different species having similar structure of hands is an example of biodiversity.
  2. Limbs being used by different species for different kinds of work is an example of biodiversity.
  3. Man and the aforementioned animals having similar structure of limbs is an example of coincidence in evolution.
  4. Man and the aforementioned animals have a shared evolutionary history.

Passage – 7

Around 56 million years ago, the Atlantic Ocean had not fully opened and animals, perhaps including our primate ancestors, could walk from Asia to North America through Europe and across Greenland. Earth was warmer than it is today, but as the Palaeocene epoch gave way to Eocene, it was about to get much warmer still —rapidly and radically. The cause was a massive geologically sudden release of carbon. During this period called Palaeocene - Eocene Thermal Maximum or PETM, the carbon injected into the atmosphere was roughly the amount that would be injected today if humans burned all the Earth's reserves of coal, oil and natural gas. The PETM lasted for about 1,50,000 years, until the excess carbon was reabsorbed. It brought on drought, floods, insect plagues and a few extinctions. Life on Earth survived — indeed, it prospered — but it was drastically different.

Q. Based on the above passage, the following assumptions have been made:

  1. Global warming has a bearing on the planet's biological evolution.
  2. Separation of land masses causes the release of huge quantities of carbon into the atmosphere.
  3. Increased warming of Earth's atmosphere can change the composition of its flora and fauna.
  4. The present man-made global warming will finally lead to conditions similar to those which happened 56 million years ago.

Which of the assumptions given above are valid?

  1. 1 and 2
  2. 3 and 4
  3. 1 and 3
  4. 2 and 4

Passage –8

Low-end IoT (Internet of Things) devices are cheap commodity items: addressing security would add to the cost. This class of items is proliferating with new applications; many home appliances, thermostats, security and monitoring devices and personal convenience devices are part of the IoT. So are fitness trackers, certain medical implants and computer-like devices in automobiles. The IoT is expected to expand exponentially — but new security challenges are daunting.

Q. Which one of the following statements is the most logical and rational inference that can be made from the above passage?

  1. Development of enabling technologies in India can be a big boost to its manufacturing sector.
  2. India is not yet fully ready to adopt IoT in view of the imminent security challenges.
  3. Life becomes more comfortable with the development of cheap low-end IoT devices.
  4. As we go digital, we must recognise the huge threat to Internet security from some IoT devices.

Passage – 9

With the digital phenomenon restructuring most social sectors, it is little surprise that global trade negotiations are now eyeing the digital area in an attempt to pre-emptively colonise it. Big Data is freely collected or mined from developing countries, and converted into digital intelligence in developed countries. This intelligence begins to control different sectors and extract monopoly rents. A large foreign company providing cab service, for instance, is not a work of cars and drivers; it is digital intelligence about commuting, public transport, roads, traffic, city events, personal behavioural characteristics of commuters and driver and so on.

Q. Which one of the following is the most logical and rational corollary to the above passage?

  1. Globalization is not in the interests of India as it undermines its socio-economic structures.
  2. India should be careful to protect its digital sovereignty in global trade talks.
  3. India should charge monopoly rents from multinational companies in exchange for Big Data.
  4. The loss of Big Data from India is proportional to the degree/value of its foreign trade.

Q. Which of the following is most definitively implied by the above passage?

  1. Big Data is the key resource in the digital space.
  2. Big economies create Big Data.
  3. Access to Big Data is the prerogative of developed countries.
  4. Access to and possession of Big Data is a characteristic of developed countries.

Passage – 10

The rural poor across the world, including India, have contributed little to human-induced climate change, yet they are on the frontline in coping with its effects. Farmers can no longer rely on historical averages for rainfall and temperature, and the more frequent and extreme weather events, such as droughts and floods, can spell disaster. And there are new threats, such as sea level rise and the impact of melting glaciers on water supply. How significant are small farms? As many as two billion people worldwide depend on them for their food and livelihood. Small-holder farmers in India produce 41 percent of the country's food grains, and other food items that contribute to local and national food security.

Q. What is the most logical and rational corollary to the above passage?

  1. Supporting small farmers is an important part of any agenda regarding environmentally sustainable development.
  2. Poor countries have little role to play in the mitigation of global warming.
  3. Due to a large number of farmer households, India will not have food security problem in the foreseeable future.
  4. Only small-holder farmers in India can ensure food security.

Q. The above passage implies that

  1. There is a potential problem of food insecurity in India.
  2. India will have to strengthen its disaster management capabilities.

Which of the above assumptions is/are valid?

  1. 1 only
  2. 2 only
  3. Both 1 and 2
  4. Neither 1 nor 2

Passage – 11

A changing climate, and the eventual efforts of governments (however reluctant) to deal with it, could have a big impact on investors' returns. Companies that produce or use large amounts of fossil fuels will face higher taxes and regulatory burdens. Some energy producers may find it impossible to exploit their known reserves, and be left with "stranded assets" — deposits of oil and coal that have to be left in the ground. Other industries could be affected by the economic damage caused by more extreme weather — storms, floods, heat waves and droughts.

Q. On the basis of the above passage, the following assumptions have been made:

  1. Governments and companies need to be adequately prepared to face the climate change.
  2. Extreme weather events will reduce the economic growth of governments and companies in future.
  3. Ignoring climate change is a huge risk for investors.

Which of the above assumptions is/are valid?

  1. 1 and 2 only
  2. 3 only
  3. 1 and 3 only
  4. 1, 2 and 3

Passage – 12

Access to schooling for those coming of school age is close to universal, but access to quality exhibits a sharp gradient with socio-economic status. Quotas for the weaker sections in private schools is a provision introduced by the Right of Children to Free and Compulsory Education Act, 2009. The quotas have imposed a debate on issues of social integration and equity in education that private actors had escaped by and large. The idea of egalitarian education system with equality of opportunity as its primary goal appears to be outside the space that private school principals inhabit. Therefore, the imposition of the quotas has led to resistance, sometimes justified.

Q. With reference to the above passage, the following assumptions have been made:

  1. Making equality of opportunity a reality is the fundamental goal of the Indian education system.
  2. The present Indian school system is unable to provide egalitarian education.
  3. Abolition of private schools and establishment of more government schools is the only way to ensure egalitarian education.

Which of the above assumptions is/are valid?

  1. 1 and 2 only
  2. 2 only
  3. 2 and 3 only
  4. 3 only

Passage – 13

A majority of the TB infected in India are poor and lack sufficient nutrition, suitable housing and have little understanding of prevention. TB then devastates families, makes the poor poorer, particularly affects women and children, and leads to ostracisation and loss of employment. The truth is that even if TB does not kill them, hunger and poverty will. Another truth is that deep-seated stigma, lack of counselling, expensive treatment and lack of adequate support from providers and family, coupled with torturous side-effects demotivate patients to continue treatment — with disastrous health consequences.

Q. Which one of the following is the most logical, rational and crucial message conveyed by the above passage?

  1. TB is not a curable disease in Indian circumstances.
  2. Curing TB requires more than diagnosis and medical treatment.
  3. Government's surveillance mechanism is deficient; and poor people have no access to treatment.
  4. India will be free from diseases like TB only when its poverty alleviation programmes are effectively and successfully implemented.

Passage — 14

What stands in the way of the widespread and careful adoption of 'Genetic Modification (GM)' technology is an `Intellectual Property Rights' regime that seeks to create private monopolies for such technologies. If GM technology is largely corporate driven, it seeks to maximize profits and that too in the short run. That is why corporations make major investments for herbicide-tolerant and pest-resistant crops. Such properties have only a short window, as soon enough pests and weeds will evolve to overcome such resistance. This suits the corporations. The National Farmers Commission pointed out that priority must be given in genetic modification to the incorporation of genes that can help impart resistance to drought, salinity and other stresses.

Q. Which one of the following is the most logical, rational and crucial message conveyed by the above passage?

  1. Public research institutions should take the lead in GM technology and prioritise the technology agenda.
  2. Developing countries should raise this issue in WTO and ensure the abolition of Intellectual Property Rights.
  3. Private corporations should not be allowed to do agribusiness in India, particularly the seed business.
  4. Present Indian circumstances do not favour the cultivation of genetically modified crops.

Q. On the basis of the above passage, the following assumptions have been made:

  1. The issue of effects of natural calamities on agriculture is not given due consideration by GM technology companies.
  2. In the long run, GM technology will not be able to solve agricultural problems arising due to global warming.

Which of the above assumptions is/are valid?

  1. 1 only
  2. 2 only
  3. Both 1 and 2
  4. Neither 1 nor 2

Passage – 15

Most invasive species are neither terribly successful nor very harmful. Britain's invasive plants are not widespread, not spreading especially quickly, and often less of a nuisance than vigorous natives such as bracken. The arrival of new species almost always increases biological diversity in a region; in many cases, a flood of newcomers drives no native species to extinction. One reason is that invaders tend to colonise disturbed habitats like polluted lakes and post-industrial wasteland, where little else lives. They are nature's opportunists.

Q. Which one of the following is the most logical and rational inference that can be made from the above passage?

  1. Invasive species should be used to rehabilitate desert areas and wastelands of a country.
  2. Laws against the introduction of foreign plants are unnecessary.
  3. Sometimes, the campaigns against foreign plants are pointless.
  4. Foreign plants should be used to increase the biodiversity of a country.

Passage – 16

Diarrhoeal deaths among Indian children are mostly due to food and water contamination. Use of contaminated groundwater and unsafe chemicals in agriculture, poor hygiene in storage and handling of food items to food cooked and distributed in unhygienic surroundings; there are myriad factors that need regulation and monitoring. People need to have awareness of adulteration and ways of complaining to the relevant authorities. Surveillance of food-borne diseases involves a number of government agencies and entails good training of inspection staff. Considering the proportion of the urban population that depends on street food for its daily meals, investing in training and education of street vendors is of great significance.

Q. On the basis of the above passage, the following assumptions have been made:

  1. Food safety is a complex issue that calls for a multipronged solution.
  2. Great investments need to be made in developing the manpower for surveillance and training.
  3. India needs to make sufficient legislation for governing food processing industry.

Which of the above assumptions is/are valid?

  1. 1 and 2 only
  2. 3 only
  3. 1 and 3 only
  4. 1, 2 and 3

Passage – 17

The interests of working and poor people have historically been neglected in the planning of our cities. Our cities are increasingly intolerant, unsafe and unlivable places for large numbers of citizens and yet we continue to plan via the old ways — the static Development Plan — that draws exclusively from technical expertise, distanced from people's live experiences and needs, and actively excluding large number of people, places, activities and practices that are an integral part of the city.

Q. The passage seems to argue

  1. against the monopoly of builders and the interests of elite groups.
  2. against the need for global and smart cities.
  3. in favour of planning cities mainly for working class and poor people.
  4. in favour of participation of peoples' groups in city planning.

Passage – 18

A vast majority of Indians are poor, with barely 10 percent employed in the organised sector. We are being convinced that vigorous economic growth is generating substantial employment. But this is not so. When our economy was growing at 3 percent per year, employment in the organised sector was growing at 2 percent per year. As the economy began to grow at 7 - 8 percent per year, the rate of growth of employment in the organised sector actually declined to 1 percent per year.

Q. The above passage seems to imply that

  1. most of modern economic growth is based on technological progress.
  2. much of modern Indian economy does not nurture sufficient symbiotic relationship with labour-intensive, natural resource-based livelihoods.
  3. service sector in India is not very labour-intensive.
  4. literate rural population is not willing to enter organised sector.

Which of the statements given above are correct?

  1. 1 and 2 only
  2. 3 and 4 only
  3. 1, 2 and 3 only
  4. 1, 2, 3 and 4

Passage — 19

India has banking correspondents, who help bring people in the hinterland into the banking fold. For them to succeed, banks cannot crimp on costs. They also cannot afford to ignore investing in financial education and literacy. Banking correspondents are way too small to be viewed as a systemic risk. Yet India's banking regulator has restricted them to serving only one bank, perhaps to prevent arbitrage. Efforts at banking outreach may succeed only if there are better incentives at work for such last-mile workers and also those providers who ensure not just basic bank accounts but also products such as accident and life insurance and micro pension schemes.

Q. Which one of the following is the most logical, rational and crucial inference that can be derived from the above passage?

  1. Efforts to bring people in India's hinterland into the banking system are not successful.
  2. For meaningful financial inclusion, India's banking system needs more number of banking correspondents and other such last-mile workers.
  3. Meaningful financial inclusion in India requires that banking correspondents have diverse skills
  4. Better banking outreach would be impossible unless each banking correspondent is allowed to serve a number of banks

Passage – 20

India's economic footprint, given its population, still remains small compared to the US, the European Union or China. It has much to learn from other economies, yet must implement solutions that fit its unique circumstances. India especially needs an effective long-term regulatory system based on collaboration rather than the ' current top-down approach. Regulations seek desirable outcomes yet are repeatedly used as political tools to push one agenda or another. Often, regulations fail to consider impacts on jobs and economic growth — or less restrictive alternatives. Regulations may be used to protect local markets at the expense of more widely shared prosperity in the future. Additionally, regulations inevitably result in numerous unintended consequences. In today's hyper competitive global economy, regulations need to be viewed as "weapons" that seek cost-justified social and environmental benefits while improving the economic well-being of most citizens.

Q. Which one of the following is the most logical, rational and crucial inference that can be derived from the above passage?

  1. A better regulatory system will help India achieve the size of economy appropriate to its population.
  2. In a competitive global economy, India must use regulations strategically.
  3. Regulations in India do not favour its integration with today's hyper competitive global economy.
  4. Job creation and economic growth should be dominant considerations in developing India's regulatory system.

Q. On the basis of the above passage, the following assumptions have been made:

In today's global economy,

  1. regulations are not effectively used to protect local markets.
  2. social and environmental concerns are generally ignored by the governments across the world while implementing the regulations.

Which of the above assumptions is/are valid?

  1. 1 only
  2. 2 only
  3. Both 1 and 2
  4. Neither 1 nor 2

Passage — 21

In a study, scientists compared the microbiomes of poorly nourished and well nourished infants and young children. Gut microbes were isolated from faecal samples of malnourished and healthy children. The microbiome was "immature" and less diverse in malnourished children compared to the better developed "mature" microbiome found in healthy children of the same age. According to some studies, the chemical composition of mother's milk has shown the presence of a modified sugar (sialylated oligosaccharides). This is not utilized by the baby for its own nutrition. However, the bacteria constituting the infant's microbiome thrive on this sugar which serves as their food. Malnourished mothers have low levels of this sugar in their milk. Consequently, the microbiomes of their infants fail to mature. That in turn, leads to malnourished babies.

Q. Which one of the following is the most logical, rational and crucial inference that can be derived from the above passage?

  1. If malnourished condition in children is caused by gut bacteria, it cannot be treated.
  2. The guts of malnourished babies should be inoculated with mature microbiomes.
  3. Babies of malnourished mothers should be fed with dairy milk fortified with sialylated oligosaccharides instead of mother's milk.
  4. Research on benign effects of gut bacteria on nutrition has policy implications.

Q. On the basis of the above passage, the following assumptions have been made:

  1. Processed probiotic foods are a solution to treat the children suffering from malnutrition due to immature gut bacteria composition.
  2. The babies of malnourished mothers generally tend to be' malnourished.

Which of the above assumptions is/are valid?

  1. 1 only
  2. 2 only
  3. Both 1 and 2
  4. Neither 1 nor 2

Passage – 22

Temperatures have risen nearly five times as rapidly on the Western Antarctic Peninsula than the global average over the past five decades. Researchers have now found that melting glaciers are causing a loss of species diversity among benthos in the coastal waters off the Antarctic Peninsula, Impacting an entire seafloor ecosystem. They believe increased levels of suspended sediment in water to be the cause of the dwindling biodiversity in the coastal region.

Q. On the basis of the above passage, the following assumptions have been made:

  1. Regions of glaciers warm faster than other regions due to global warming.
  2. Global warming can lead to seafloor sedimentation in some areas.
  3. Melting glaciers can reduce marine biodiversity in some areas.

Which of the above assumptions is/are valid?

  1. 1 and 2 only
  2. 3 only
  3. 2 and 3 only
  4. 1, 2 and 3

Passage -23

A research team examined a long-term owl roost. Owls prey on small mammals and the excreted remains of those meals that accumulated over the time, provide us an insight into the composition and structure of small mammals over the past 'millennia. The research suggested that when the Earth went through a period of rapid warming about 13,000 years ago, the small mammal community was stable and resilient. But, from the last quarter of the nineteenth century, human-made changes to the environment had caused an enormous drop in biomass and energy flow. This dramatic decline in energy flow means modern ecosystems are not adapting as easily as they did in the past.

Q. On the basis of the above passage, the following assumptions have been made:

  1. Global warming is a frequently occurring natural phenomenon.
  2. The impending global warming will not adversely affect small mammals.
  3. Humans are responsible for the loss of the Earth's natural resilience.

Which of the above assumptions is/are valid?

  1. 1 and 2 only
  2. 3 only
  3. 2 and 3 only
  4. 1, 2 and 3

Passage — 24

Food varieties extinction is happening all over the world --- and it is happening fast. For example, of the 7,000 apple varieties that were grown during the nineteenth century, fewer than o hundred remain. In the Philippines, thousands of varieties of rice once thrived; now only up to a hundred are grown there. In China, 90 percent of the wheat varieties cultivated just a century ago have disappeared. Farmers in the past painstakingly bred and developed crops well suited to the peculiarities of their local climate and environment. In the recent past, our heavy dependence on a few high yielding varieties and technology-driven production and distribution of food is causing the dwindling of diversity in food crops. If some mutating crop disease or future climate change decimates the few crop plants we have come to depend on to feed our growing population, we might desperately need some of those varieties we have let go extinct.

Q. On the basis of the above passage, the following assumptions have been made:

  1. Humans have been the main reason for the large scale extinction of plant species.
  2. Consumption of food mainly from locally cultivated crops ensures crop diversity.
  3. The present style of production and distribution of food will finally lead to the problem of food scarcity in the near future.
  4. Our food security may depend on our ability to preserve the locally cultivated varieties of crops.

Which of the above assumptions are valid?

  1. 1 and 3
  2. 2 and 4
  3. 2 and 3
  4. 1 and 4


Civil Services Examination 2018


Read the following passages and answer the questions that follow. Your answers to these items should be based on the passage only.

Passage-1

Global population was around 1.6 billion in 1990—today it is around 7.2 billion and growing. Recent estimates on population growth predict a global population of 9.6 billion in 2050 and 10.9 billion in 2100. Unlike Europe and North America, where only three to four per cent of population is engaged in agriculture, around 47 per cent of India's population is dependent upon agriculture. Even if India continues to do well in the service sector and the manufacturing sector picks up, it is expected that around 2030 when India overtakes China as the world's most populous country, nearly 42 per cent of India's population will still be predominantly dependent on agriculture.

Q. Which of the following is the most logical and rational inference that can be made from the above passage?

  1. Prosperity of agriculture sector is of critical importance to India.
  2. Indian economy greatly depends on its agriculture.
  3. India should take strict measures to control its rapid population growth.
  4. India's farming communities should switch over to other occupations to improve their economic conditions.

Passage-2

Many pathogens that cause foodborne illnesses are unknown. Food contamination can occur at any stage from farm to plate. Since most cases of food poisoning go unreported, the true extent of global foodborne illnesses is unknown. Improvements in international monitoring have led to greater public awareness, yet the rapid globalization of food production increases consumers' vulnerability by making food harder to regulate and trace. "We have the world on our plates", says an official of WHO.

Q. Which of the following is the most logical corollary to the above passage?

  1. With more options for food come more risks.
  2. Food processing is the source of all foodborne illnesses.
  3. We should depend on locally produced food only.
  4. Globalization of food production should be curtailed.

Passage-3

I am a scientist, privileged to be somebody who tries to understand nature using the tools of science. But it is also clear that there are some really important questions that science cannot really answer, such as: Why is there something instead of nothing? Why are we here? In those domains, I have found that faith provides a better path to answers. I find it oddly anachronistic that in today's culture there seems to be a widespread presumption that scientific and spiritual views are incompatible.

Q. Which of the following is the most logical and rational inference that can be made from the above passage?

  1. It is the faith and not science that can finally solve all the problems of mankind.
  2. Science and faith can be mutually complementary if their proper domains are understood.
  3. There are some very fundamental questions which cannot be answered by either science or faith.
  4. In today's culture, scientific views are given more importance than spiritual views.

Passage-4

Though I have discarded much of past tradition and custom, and am anxious that India should rid herself of all shackles that bind and contain her and divide her people, and suppress vast numbers of them, and prevent the free development of the body and the spirit; though I seek all this, yet I do not wish to cut myself off from that past completely. I am proud of that great inheritance that has been and is, ours and I am conscious that I too, like all of us, am a link in that unbroken chain which goes back to the dawn of history in the immemorial past of India.

Q. The author wants India to rid herself of certain past bonds because

  1. he is not able to see the relevance of the past
  2. there is not much to be proud of
  3. he is not interested in the history of India
  4. they obstruct her physical and spiritual growth

Passage-5

It is no longer enough for us to talk about providing for universal access to education. Making available schooling facilities is an essential prerequisite, but is insufficient to ensure that all children attend school and participate in the learning process. The school may be there, but children may not attend or they may drop out after a few months. Through school and social mapping, we must address the entire gamut of social, economic, cultural and indeed linguistic and pedagogic issues, factors that prevent children from weaker sections and disadvantaged groups, as also girls, from regularly attending and complementing elementary education. The focus must be on the poorest and most vulnerable since these groups are the most disempowered and at the greatest risk of violation or denial of their right to education. The right to education goes beyond free and compulsory education to include quality education for all. Quality is an integral part of the right to education. If the education process lacks quality, children are being denied their right. The Right of Children to Free and Compulsory Education Act lays down that the curriculum should provide for learning through activities, exploration and discovery. This places an obligation on us to change our perception of children as passive receivers of knowledge, and to move beyond the convention of using textbooks as the basis of examinations. The teaching-learning process must become stress-free, and a massive programme for curricular reform should be initiated to provide for a child-friendly learning system, that is more relevant and empowering. Teacher accountability systems and processes must en sure that children are learning, and that their right to learn in a child-friendly environment is not violated. Testing and assessment systems must be reexamined and redesigned to ensure that these do not force children to struggle between school and tuition centres, and bypass childhood.

Q. According to the passage, which of the following is/are of paramount importance under the Right to Education?

  1. Sending of children to school by all parents
  2. Provision of adequate physical infrastructure in schools
  3. Curricular reforms for developing child-friendly learning system

Select the correct answer using the code given below:

  1. 1 only
  2. 1 and 2 only
  3. 3 only
  4. None of the above

Q. With reference to the above passage, the following assumptions have been made:

  1. The Right to Education guarantees teachers’ accountability for the learning process of children.
  2. The Right to Education guarantees 100% enrolment of children in the schools.
  3. The Right to Education intends to take full advantage of demographic dividend.

Which of the above assumptions is/ are valid?

  1. 1 only
  2. 2 and 3 only
  3. 3 only
  4. 1, 2 and 3

Q. According to the passage, which one of the following is critical in bringing quality in education?

  1. Ensuring regular attendance of children as well as teachers in school
  2. Giving pecuniary benefits to teachers to motivate them
  3. Understanding the socio cultural background of children
  4. Inculcating learning through activities and discovery

Q. What is the essential message in this passage?

  1. The Right to Education now is a Fundamental Right.
  2. The Right to Education enables the children of poor and weaker sections of the society to attend schools.
  3. The Right to Free and Compulsory Education should include quality education for all.
  4. The Government as well as parents should ensure that all children attend schools.

Passage-6

`Desertification’ is a term used to explain a process of decline in the biological productivity of an ecosystem, leading to total loss of productivity. While this phenomenon is often linked to the arid, semi-arid and sub-humid ecosystems, even in the humid tropics, the impact could be most dramatic. Impoverishment of human-impacted terrestrial ecosystems may exhibit itself in a variety of ways: accelerated erosion as in the mountain regions of the country, salinization of land as in the semi-arid and arid ‘green revolution’ areas of the country, e.g., Haryana and western Uttar Pradesh, and site quality decline—a common phenomenon due to general decline in tree cover and monotonous monoculture of rice/wheat across the Indian plains. A major consequence of deforestation is that it relates to adverse alterations in the hydrology and related soil and nutrient losses. The consequences of deforestation invariably arise out of site degradation through erosive losses. Tropical Asia, Africa and South America have the highest levels of erosion. The already high rates for the tropics are increasing at an alarming rate (e.g., through the major river systems— Ganga and Brahmaputra, in the context), due to deforestation and land management practices subsequent to forest clearing. In the mountain context, the declining moisture retention of the mountain soils, drying up of the ‘underground springs and smaller rivers in the Himalayan could be attributed to drastic changes the forest cover. An indirect consequence is drastic alteration in the upland-lowland interaction, mediated through water. The current concern the tea planter of Assam has is about the damage to tea plantations due to frequent inundation along the flood-plains of Brahmaputra, and the damage to tea plantation and the consequent loss in tea productivity is due to rising level of the river bottom because of siltation and the changing course of the river system. The ultimate consequences of site desertification are soil degradation, alteration in available water’ and its quality, and the consequent decline in food, fodder and fuel-wood essential for the economic well-being of rural communities.

Q. According to the passage, which of the following are the consequences of decline in forest cover?

  1. Loss of topsoil
  2. Loss of smaller rivers
  3. Adverse effect on production
  4. Declining of groundwater

Select the correct answer using the code given below:

  1. 1, 2 and 3 only
  2. 2, 3 and 4 only
  3. 1 and 4 only
  4. 1, 2, 3 and 4

Q. Which of the following is/are the correct inference/ inferences that can be made from the passage?

  1. Deforestation can cause changes in the course of rivers.
  2. Salinization of land takes place to human activities only.
  3. Intense monoculture practice in plains is a major reason for desertification in Tropical Asia, Africa and South America.

Select the correct answer using the code given below:

  1. 1 only
  2. 1 and 2 only
  3. 2 and 3 only
  4. None of the above is a correct inference

Q. With reference to ‘desertification’, as described in the passage, the following assumptions have been made:

  1. Desertification is a phenomenon in tropical areas only.
  2. Deforestation invariably leads to floods and desertification.

Which of the above assumptions is/ are valid?

  1. 1 only
  2. 2 only
  3. Both 1 and 2
  4. Neither 1 nor 2

Passage-7

A diversity of natural assets will be needed to cope with climate change and ensure productive agriculture, forestry, and fisheries. For example, crop varieties are needed that perform well under drought, heat, and enhanced CO2. But the private sector and farmer-led process of choosing crops favours homogeneity adapted to past or current conditions, not varieties capable of producing consistently high yields in warmer, wetter, or drier conditions. Accelerated breeding programmes are needed to conserve a wider pool of genetic resources of existing crops, breeds, and their wild relatives. Relatively intact ecosystems, such as forested catchments, mangroves, wetlands, can buffer the impacts of climate change. Under a changing climate, these ecosystems are themselves at risk, and management approaches will need to be more proactive and adaptive. Connections between natural areas, such as migration corridors, may be needed to facilitate species movements to keep up with the change in climate.

Q. With reference to the above passage, which of the following would assist us in coping with the climate change?

  1. Conservation of natural water sources
  2. Conservation of wider gene pool
  3. Existing crop management practices
  4. Migration corridors

Select the correct answer using the code given below:

  1. 1, 2 and 3 only
  2. 1, 2 and 4 only
  3. 3 and 4 only
  4. 1, 2, 3 and 4

Q. With reference to the above passage, the following assumptions have been made:

  1. Diversification of livelihoods acts as a coping strategy for climate change.
  2. Adoption of monocropping practice leads to the extinction of plant varieties and their wild relatives.

Which of the above assumptions is/are valid?

  1. 1 only
  2. 2 only
  3. Both 1 and 2
  4. Neither 1 nor 2

Passage-8

Today, the top environmental challenge is a combination of people and their aspirations. If the aspirations are more like the frugal ones we had after the Second World War, a lot more is possible than if we view the planet as a giant shopping mall. We need to get beyond the fascination with glitter and understand that the planet works as a biological system.

Q. Which of the following is the most crucial and logical inference that can be made from the above passage?

  1. The Earth can meet only the basic needs of humans for food, clothing and shelter.
  2. The only way to meet environmental challenge is to limit human population.
  3. Reducing our consumerism is very much in our own interest.
  4. Knowledge of biological systems can only help us save this planet.

Passage-9

Some people believe that leadership is a quality which you have at birth or not at all. This theory is false, for the art of leadership can be acquired and can indeed be taught. This discovery is made in time of war and the results achieved can surprise even the instructors. Faced with the alternatives of going left or right, every soldier soon grasps that a prompt decision either way is better than an endless discussion. A firm choice of direction has an even chance of being right while to do nothing will be almost certainly wrong.

Q. The author of the passage holds the view that

  1. leadership can be taught through war experience only
  2. leadership can be acquired as well as taught
  3. the results of training show that more people acquire leadership than are expected
  4. despite rigorous instruction, very few leaders are produced

Passage-10

All actions to address climate change ultimately involve costs. Funding is vital in order for countries like India to design and implement adaptation and mitigation plans and projects. The problem is more severe for developing countries like India, which would be one of the hardest hit by climate change, given its need to finance development. Most countries do indeed treat climate change as real threat and are striving to address it in a more comprehensive and integrated manner with the limited resources at their disposal.

Q. With reference to the above passage, the following assumptions have been made:

  1. Climate change is not a challenge for developed countries.
  2. Climate change is a complex policy issue and also a development issue for many countries.
  3. Ways and means of finance must be fount to enable developing countries to enhance their adaptive capacity.

Which of the above assumptions is/are valid?

  1. 1 and 2 only
  2. 3 only
  3. 2 and 3 only
  4. 1, 2 and 3

Passage-11

Cooking with biomass and coal in India is now recognized to cause major health problems, with women and children in poor populations facing the greatest risk. There are more than 10 lakh premature deaths each year from household air pollution due to polluting cooking fuels with another 1.5 lakh due to their contribution to general outdoor air pollution in the country. Although the fraction of the Indian population using clean cooking fuels, such as LPG, natural gas and electricity, is slowly rising, the number using polluting solid fuels as their primary cooking fuel has remained static for nearly 30 years at about 70 crore.

Q. Which of the following is the most crucial and logical inference that can be made from the above passage?

  1. Rural people are giving up the use of polluting solid fuels due to their increasing awareness of health hazards.
  2. Subsidizing the use of clean cooking fuels will solve the problem of India's indoor air pollution.
  3. India should increase its import of natural gas and produce more electricity.
  4. Access to cooking gas can reduce premature deaths in poor households.

Passage-12

Scientific knowledge has its dangers, but so has every great thing. Over and beyond the dangers with which it threatens the present, it opens up as nothing else can, the vision of a possible happy world; a world without poverty, without war, with little illness. Science, whatever unpleasant consequences it may have by the way, is in its very nature a liberator.

Q. Which one of the following is the most important implication of the passage?

  1. A happy world is a dream of science.
  2. Science only can build a happy world, but it is also the only major threat.
  3. A happy world is not possible without science.
  4. A happy world is not at all possible with or without science.

Passage-13

The Arctic's vast reserves of fossil fuel, fish and minerals are now accessible for a longer period in a year. But unlike Antarctica, which is protected from exploitation by the Antarctic Treaty framed during the Cold War and is not subject to territorial claims by any country, there is no legal regime protecting the Arctic from industrialization, especially at a time when the world craves for more and more resources. The distinct possibility of ice-free summer has prompted countries with Arctic coastline to scramble for great chunks of the melting ocean.

Q. Which one of the following is the most important implication of the passage?

  1. India can have territorial claims in the Arctic territory and free access to its resources.
  2. Melting of summer ice in the Arctic leads to changes in the geopolitics.
  3. The Arctic region will solve the world's future problem of resource crunch.
  4. The Arctic region has more resources than Antarctica.

Passage-14

Being a member of the WTO, India is bound by the agreements that have been signed and ratified by its members, including itself. According to Article 6 of the Agriculture Agreement, providing minimum support prices for agricultural products is considered distorting and is subject to limits. The subsidy arising from 'minimal supports' cannot exceed 10 per cent of the value of agricultural production for developing countries. PDS in India entails minimum support prices and public stockholding of food grains. It is possible that, in some years, the subsidy to producers will exceed 10 per cent of the value of agricultural production.

Q. What is the crucial message conveyed by the above passage?

  1. India should revise its PDS.
  2. India should not be a member of WTO.
  3. For India, food security collides with trade.
  4. India provides food security to its poor.

Passage-15

India's educational system is modelled on the mass education system that developed in the 19th century in Europe and later spread around the world. The goal of the system is to condition children as 'good' citizens and productive workers. This suited the industrial age that needed the constant supply of a compliant workforce with a narrow set of capabilities. Our educational institutes resemble factories with bells, uniforms and batch-processing of learners, designed to get learners to conform. But, from an economic point of view, the environment today is very different. It is a complex, volatile and globally interconnected world.

Q. With reference to the above passage, the following assumptions have been made:

  1. India continues to be a developing country essentially due to its faulty education system.
  2. Today's learners need to acquire new-age skill-sets.
  3. A good number of Indians go to some developed countries for education because the educational systems there are a perfect reflection of the societies in which they function.

Which of the above assumptions is/are valid?

  1. 1 and 3 only
  2. 2 only
  3. 2 and 3 only
  4. 1, 2 and 3

Passage-16

The practice of dieting has become an epidemic; everyone is looking out for a way to attain that perfect body. We are all different with respect to our ethnicity, genetics, family history, gender, age, physical and mental and spiritual health status, lifestyles and preferences. Thereby we also differ in what foods we tolerate or are sensitive to. So we really cannot reduce so many complexities into one diet or diet book. This explains the failure of diets across the world in curbing obesity. Unless the reasons for weight gain are well understood and addressed and unless habits are changed permanently, no diet is likely to succeed.

Q. What is the most logical and rational inference that can be made from the above passage?

  1. Obesity has become an epidemic all over the world.
  2. A lot of people are obsessed with attaining a perfect body.
  3. Obesity is essentially an incurable disease.
  4. There is no perfect diet or one solution for obesity.

Passage-17

Monoculture carries great risks. A single disease or pest can wipe out swathes of the world's food production, an alarming prospect given that its growing and wealthier population will eat 70% more by 2050. The risks are magnified by the changing climate. As the planet warms and monsoon rains intensify, farmlands in Asia will flood. North America will suffer more intense droughts, and crop diseases will spread to new latitudes.

Q. Which of the following is the most logical, rational and crucial message given by the passage?

  1. Preserving crop genetic diversity is an insurance against the effects of climate change.
  2. Despite great risks, monoculture is the only way to ensure food security in the world.
  3. More and more genetically modified crops only can save the world from impending shortages of food.
  4. Asia and North America will be worst sufferers from climate change and the consequent shortage of food.

Passage-18

The quest for cheap and plentiful meat has resulted in factory farms where more and more animals are squeezed into smaller lots in cruel and shocking conditions. Such practices have resulted in many of the world’s health pandemics such as the avian flu. Worldwide, livestock are increasingly raised in cruel, cramped conditions, where animals spend their short lives under artificial light, pumped full of antibiotics and growth hormones, until the day they are slaughtered. Meat production is water intensive. 15000 litres of water is needed for every kilogram of meat compared with 3400 litres for rice, 3300 litres for eggs and 255 litres for a kilogram of potatoes.

Q. What is the most rational and crucial message given by the passage?

  1. Mass production of meat through industrial farming is cheap and is suitable for providing protein nutrition to poor countries.
  2. Meat-producing industry violates the laws against cruelty to animals.
  3. Mass production of meat through industrial farming is undesirable and should be stopped immediately.
  4. Environmental cost of meat production is unsustainable when it is produced through industrial farming.

Passage-19

A male tiger was removed from Pench Tiger Reserve and was relocated in Panna National Park. Later, this tiger trekked toward his home 250 miles away. The trek of this solitary tiger highlights a crisis. Many wildlife reserves exist as islands of fragile habitat in a vast sea_of humanity, yet tigers can range over a hundred miles, seeking prey, mates and territory. Nearly a third of India’s tigers live outside tiger reserves, a situation that is_dangerous for both human and animal. ‘Prey and tigers can only disperse if there are recognized corridors of land/ between protected areas to allow unmolested passage.

Q. Which of the following is the most rational and crucial message given by the passage?

  1. The conflict between man and wildlife cannot be resolved, no matter what efforts we make.
  2. Safe wildlife corridors between protected areas is an essential aspect of conservation efforts.
  3. India needs to declare more protected areas and set up more tiger reserves.
  4. India’s National Parks and Tiger Reserves need to be professionally managed.

Q. With reference to the above passage, the following assumptions have been made:

  1. The strategy of conservation of wildlife by relocating them from one protected area to another is not often successful.
  2. India does not have suitable legislation to save the tigers, and its conservation efforts have failed which forced the tigers to live outside protected areas.

Which of the above assumptions is/are valid?

  1. 1 only
  2. 2 only
  3. Both 1 and 2
  4. Neither 1 nor 2


Civil Services Examination 2017


Read the following passages and answer the questions that follow. Your answers to these items should be based on the passage only.

Passage-1

Disruption of traditional institutions, identifications and loyalties is likely to lead to ambivalent situations. It is possible that some people may renew their identification with traditional groups whereas others align themselves with new groups and symbols emergent from processes of political development. In addition, political development tends to foster group awareness of a variety of class, tribe, region, clan, language, religion, occupation and others.

Q. Which one of the following is the best explanation of the above passage?

  1. Political development is not a unilinear process for it involves both growth and decay.
  2. Traditional societies succeed in resisting positive aspects of political development.
  3. It is impossible for traditional societies to break away from lingering loyalties.
  4. Sustenance of traditional loyalties is conducive to political development.

Passage-2

There has been a significant trend worldwide towards regionalism in government, resulting in a widespread transfer of powers downwards towards regions and communities since 1990s. This process, which involves the creation of new political entities and bodies at a sub-national level and an increase in their content and powers, is known as devolution. Devolution has been characterized as being made up of three factors—political legitimacy, decentralization of authority and decentralization of resources. Political legitimacy here means a mass demand from below for the decentralization process, which is able to create a political force for it to take place. In many cases, decentralization is initiated by the upper tier of government without sufficient political mobilization for it at the grassroots level, and in such cases the decentralization process often does not fulfil its objectives.

Q. Which among the following is the most logical, rational and critical inference that can be made from the above passage?

  1. Emergence of powerful mass leaders is essential to create sub-national political entities and thus ensure successful devolution and decentralization.
  2. The upper tier of government should impose devolution and decentralization on the regional communities by law or otherwise.
  3. Devolution, to be successful, requires a democracy in which there is free expression of the will of the people at lower level and their active participation at the grassroots level.
  4. For devolution to take place, a strong feeling of regionalism in the masses is essential.

Passage-3

We live in digital times. The digital is not just something we use strategically and specifically to do a few tasks. Our very perception of who we are, how we connect to the world around us, and the ways in which we define our domains of life, labour and language are hugely structured by the digital technologies. The digital is everywhere and; like air, invisible. We live within digital systems, we live with intimate gadgets, we interact through digital media, and the very presence and imagination of the digital has dramatically restructured our lives. The digital, far from being a tool, is a condition and context that defines the shapes and boundaries of our understanding of the self, the society, and the structure of governance.

Q. Which among the following is the most logical and essential message conveyed by the above passage?

  1. All problems of governance can be solved by using digital technologies.
  2. Speaking of digital technologies is speaking of our life and living.
  3. Our creativity and imagination cannot be expressed without digital media.
  4. Use of digital systems is imperative for the existence of mankind in future.

Passage-4

The IMF has pointed out that the fast growing economies of Asia face the risk of falling into 'middle-income trap'. It means that average incomes in these countries, which till now have been growing rapidly, will stop growing beyond a point—a point that is well short of incomes in the developed West. The IMF identifies a number of causes of middle-income trap—none of which is surprising—from infrastructure to weak institutions, to less than favourable macroeconomic conditions. But the broad, overall cause, says IMF, is a collapse in the growth of productivity.

Q. Which among the following is the most logical, rational and critical inference that can be made from the above passage?

  1. Once a country reaches middle-income stage, it runs the risk of falling productivity which leads to stagnant incomes.
  2. Falling into middle-income trap is a general characteristic of fast growing economies.
  3. There is no hope at all for emerging Asian economies to sustain the growth momentum.
  4. As regards growth of productivity, the performance of Asian economies is not satisfactory.

Passage-5

An innovative India will be inclusive as well as technologically advanced, improving the lives of all Indians. Innovation and R&D can mitigate increases in social inequality and relieve the pressures created by rapid urbanization. The growing divergence in productivity between agriculture and knowledge-intensive manufacturing and services threatens to increase income inequality. By encouraging India's R&D labs and universities to focus on the needs of poor people and by improving the ability of informal firms to absorb knowledge, an innovation and research agenda can counter this effect. Inclusive innovation can lower the cost of goods and services and create income - earning opportunities for the poor people.

Q. Which among the following is the most logical and rational assumption that can be made from the above passage?

  1. Innovation and R&D is the only way to reduce rural to urban migration.
  2. Every rapidly growing country needs to minimize the divergence between productivity in agriculture and other sectors.
  3. Inclusive innovation and R&D can help create an egalitarian society.
  4. Rapid urbanization takes place only when a country's economic growth is rapid.

Passage-6

Climate change is likely to expose a large number of people to increasing environmental risks forcing them to migrate. The international community is yet to recognize this new category of migrants. There is no consensus on the definition and status of climate refugees owing to the distinct meaning the term refugees carry under international laws. There are still gaps in understanding how climate change will work as the root cause of migration. Even if there is recognition of climate refugees, who is going to provide protection? More emphasis has been given to international migration due to climate change. But there is a need to recognize the migration of such people within the countries also so that their problems can be addressed properly.

Q. Which of the following is the most rational inference from the above passage?

  1. The world will not be able to cope with large scale migration of climate refugees.
  2. We must find the ways and means to stop further climate change.
  3. Climate change will be the most important reason for the migration of people in the future.
  4. Relation between climate change and migration is not yet properly understood.

Passage-7

Many farmers use synthetic pesticides to kill infesting insects. The consumption of pesticides in some of the developed countries is touching 3000 grams/hectare. Unfortunately, there are reports that these compounds possess inherent toxicities that endanger, the health of the farm operators, consumers and the environment. Synthetic pesticides are generally persistent in environment. Entering in food chain they destroy the microbial diversity and cause ecological imbalance. Their indiscriminate use has resulted in development of resistance among insects to insecticides, upsetting of balance in nature and resurgence of treated populations. Natural pest control using the botanical pesticides is safer to the user and the environment because they break down into harmless compounds within hours or days in the presence of sunlight. Plants with pesticidal properties have been in nature for millions of years without any ill or adverse effects on the ecosystem. They are easily decomposed by many microbes common in most soil. They help in the maintenance of biological diversity, of predators and the reduction of environmental contamination and human health hazards. Botanical pesticides formulated from plants are biodegradable and their use in crop protection is a practical sustainable alternative.

Q. On the basis of the above passage, the following assumptions have been made:

  1. Synthetic pesticides should never be used in modem agriculture.
  2. One of the aims of sustainable agriculture is to ensure minimal ecological imbalance.
  3. Botanical pesticides are more effective as compared to synthetic pesticides.

Which of the assumptions given above is/are correct?

  1. 1 and 2 only
  2. 2 only
  3. 1 and 3 only
  4. 1, 2 and 3

Q. Which of the following statements is/are correct regarding biopesticides?

  1. They are not hazardous to human health.
  2. They are persistent in environment.
  3. They are essential to maintain the biodiversity of any ecosystem.

Select the correct answer using the code given below.

  1. 1 only
  2. 1 and 2 only
  3. 1 and 3 only
  4. 1, 2 and 3

Passage-8

An air quality index (AQI) is a way to combine measurements of multiple air pollutants into a single number or rating. This index is ideally kept constantly updated and available in different places. The AQI is most useful when lots of pollution data are being gathered and when pollution levels are normally, but not always, low. In such cases, if pollution levels spike for a few days, the public can quickly take preventive action (like staying indoors) in response to an air quality warning. Unfortunately, that is not urban India. Pollution levels in many large Indian cities are so high that they remain well above any health or regulatory standard for large part of the year. If our index stays in the Red/Dangerous' region day after day, there is not much any one can do, other than getting used to ignoring it.

Q. Which among the following is the most logical and rational inference that can be made from the above passage?

  1. Our governments are not responsible enough to keep our cities pollution free.
  2. There is absolutely no need for air quality indices in our country.
  3. Air quality index is not helpful to the residents of many of our large cities.
  4. In every city, public awareness about pollution problems should increase.

Passage-9

Productive jobs are vital for growth and a good lob is the best form of inclusion. More than half of our population depends on agriculture, but the experience of other countries suggests that the number of people dependent on agriculture will have to shrink if per capita incomes in agriculture are to go up substantially. While industry is creating jobs, too many such jobs are low-productivity non-contractual jobs in the unorganized sector, offering low incomes, little protection, and no benefits. Service jobs are relatively of high productivity, but employment growth in services has been slow in recent years.

Q. Which among the following is the most logical and rational inference that can be made from the above passage?

  1. We must create conditions for the faster growth of highly productive service jobs to ensure employment growth and inclusion.
  2. We must shift the farm workers to the highly productive manufacturing and service sectors to ensure the economic growth and inclusion.
  3. We must create conditions for the faster growth of productive jobs outside of agriculture even while improving the productivity of agriculture.
  4. We must emphasize the cultivation of high-yielding hybrid varieties and genetically modified crops to increase the per capita income in agriculture.

Passage-10

A Landscape-scale approach to land use can encourage greater biodiversity outside protected areas. During hurricane 'Mitch' in 1998, farms using eco-agricultural practices suffered 58 percent, 70 percent and 99 percent less damage in Honduras, Nicaragua and Guatemala, respectively, than farms using conventional techniques. In Costa, vegetative windbreaks and fencerows boosted farmers' income from pasture and coffee while also increasing bird diversity. Bee pollination is more effective when agricultural fields are closer to natural or semi-natural habitat, a finding that matters because 87 percent of the world's 107 leading crops depend on animal pollinators. In Costa Rica, Nicaragua and Colombia silvopastoral systems t tint integrate trees with pastureland are improving the sustainability of cattle production, and diversifying and increasing farmers' income.

Q. Which among the following is the most logical and rational inference that can be made from the above passage?

  1. Agricultural practices that enhance biodiversity can often increase farm output and reduce the vulnerability to disasters.
  2. All the countries of the world should be encouraged to replace ecoagriculture with conventional agriculture.
  3. Eco-agriculture should be permitted in protected areas without destroying the biodiversity there.
  4. The yield of food crops will be very high if eco-agricultural practices are adopted to cultivate them.

Passage-11

The medium term challenge for Indian manufacturing is to move from lower to higher tech sectors, from lower to higher value-added sectors, and from lower to higher productivity sectors. Medium tech industries are primarily capital intensive and resource processing; and high tech industries are mainly capital and technology intensive. In order to push the share of manufacturing in overall GDP to the projected 25 per cent, Indian manufacturing needs to capture the global market in sectors showing a rising trend in demand. These sectors are largely high technology and capital intensive.

Q. Which among the following is the most logical and rational inference that can be made from the above passage?

  1. India's GDP displays high value-added and high productivity levels in medium tech and resource processing industries.
  2. Promotion of capital and technology intensive manufacturing is not possible in India.
  3. India should push up the public investments and encourage the private investments in research and development, technology upgradation and skill development.
  4. India has already gained a great share in global markets in sectors showing a rising trend in demand.

Passage-12

Over the last decade, Indian agriculture has become more robust with record production of food grains and oilseeds. Increased procurement, consequently, has added huge of food grains in the granaries. India is one of the world's top producers of rice, wheat, milk, fruits and vegetables. India is still home the quarter of all undernourished people in the world. On an average, almost half of the total expenditure of nearly half of the households is on food.

Q. Which among the following is the most logical corollary to the above passage?

  1. Increasing the efficiency of farm to-fork value chain is necessary to reduce the poverty and malnutrition.
  2. Increasing the agricultural productivity will automatically eliminate the poverty and malnutrition in India.
  3. India's agricultural productivity is already great and it is not necessary to increase it further.
  4. Allocation of more funds for social welfare and poverty alleviation programmes will ultimately eliminate the poverty and malnutrition in India.

Passage-13

The States are like pearls and the Centre is the thread which turns them into a necklace; if the read snaps, the pearls are scattered.

Q. Which one of the following views corroborates the above statement?

  1. A strong Centre and strong States make the federation strong.
  2. A strong Centre is a binding force for national integrity.
  3. A strong Centre is a hindrance to State autonomy.
  4. State autonomy is a prerequisite for a federation.

Passage-14

Really I think that the poorest he that is in England has a life to live, as the greatest he, and therefore truly, I think it is clear that every man that is to live under a government ought first by his own consent to put himself under the government, and I do think that the poorest man in England is not at all bound in a strict sense to that government that he has not had a voice to put himself under.

Q. The above statement argues for

  1. distribution of wealth equally to all
  2. rule according to the consent of the governed
  3. rule of the poor
  4. expropriation of the rich

Passage-15

We have hard work ahead. There is no resting for any of us till we redeem our pledge in full fill we make all the people of India what destiny intends them to be. We are citizens of a great country, on the verge of bold advance, and we have to live up to that high standard. All of us, to whatever religion we may belong, are equally the children of India with, equal rights, privileges and obligations. We cannot encourage communalism or narrow mindedness, for no nation can be great whose people are narrow in thought or action.

Q. The challenge the author of the above passage throws to the public is to achieve

  1. a high standard of living, progress and privileges
  2. equal privileges, fulfilment of destiny and political tolerance
  3. spirit of adventure and economic parity
  4. hard work, brotherhood out national unity

Passage-16

"The individual, according to Rousseau, puts his person and all his power in common under the supreme direction of the General Will and in our corporate capacity we receive each member as an indivisible part of the whole."

Q. In the light of the above passage, the nature of General Will is best described as

  1. the sum total of the private wills of the individuals
  2. what is articulated by the elected representatives of the individuals
  3. the collective good as distinct from private wills of the individuals
  4. the material interests of the community

Passage-17

In a democratic State, where a high degree of Political maturity of the people obtains, the conflict between the will of the sovereign law-making body and the organized will of the people seldom occurs.

Q. What does the above passage imply?

  1. In a democracy, force is the main phenomenon in the actual exercise of sovereignty.
  2. In a mature democracy, force to a great extent is the main phenomenon in the actual exercise of sovereignty.
  3. In a mature democracy, use of force is irrelevant in the actual exercise of sovereignty.
  4. In a mature democracy, force is narrowed down to a marginal phenomenon in the actual exercise of sovereignty.

Passage-18

A successful democracy depends upon widespread interest and participation in politics, in which voting is an essential part. To deliberately refrain from taking such an interest, and from voting, is a kind of implied anarchy, it is to refuse one's political responsibility while enjoying the benefits of a free political society.

Q. This passage relates to

  1. duty to vote
  2. right to vote
  3. freedom to vote
  4. right to participate in politics

Passage-19

In a free country, the man who reaches the position of leader is usually one of outstanding, character and ability. Moreover, it is usually possible to foresee that he will reach such a position, since early in life one can see his qualities of character. But this is not always true in the case of a dictator; often he reaches his position of power through chance, very often through the unhappy state of his country.

Q. The passage seems to suggest that

  1. a leader foresees his future position
  2. a leader is chosen only by a free country
  3. a leader must see that his country is free from despair
  4. despair in a country sometimes leads to dictatorship

Passage-20

The greatest blessing that technological progress has in store for mankind is not, of course, an accumulation of material possessions. The amount of these that can be effectively enjoyed by one individual in one lifetime is not great. But there is not the same narrow limit to the possibilities of the enjoyment of leisure. The gift of leisure may be abused by people who have had no experience of making use of it. Yet the creative use of leisure by a minority in societies has been the mainspring of all human progress beyond the primitive level.

Q. With reference to the above passage, the following assumptions have been made:

  1. People always see the leisure time as a gift and use it for acquiring more material possessions.
  2. Use of leisure by some people to produce new and original things has been the chief source of human progress.

Which of these assumptions is/are valid?

  1. 1 only
  2. 2 only
  3. Both 1 and 2
  4. Neither 1 nor 2

Passage-21

There is more than a modicum of truth in the assertion that "a working knowledge of ancient history is necessary to the intelligent interpretation of current events". But the sage who uttered these words of wisdom might well have added something on the benefits of studying particularly the famous battles of history for the lessons they contain for those of us who lead or aspire to leadership. Such a study will reveal certain qualities and attributes which enabled the winners to win—and certain deficiemcies which caused the losers to lose and the student will see that the same pattern recurs consistently, again and again, throughout the centuries.

Q. With reference to the above passage, the following assumptions have been made:

  1. A study of the famous battles it history would help us understand the modern warfare.
  2. Studying the history is essential fin anyone who aspires to be a leader.

Which of these assumptions is/are valid?

  1. 1 only
  2. 2 only
  3. Both 1 and 2
  4. Neither 1 nor 2

Passage-22

What climate change will undeniably do is cause of amplify events that hasten the reduction of resources. Competition over these diminishing resources would ensue in the form of political or even violent conflict. Resource based conflicts have rarely been overt and are thus difficult to isolate. Instead they take on veneers that appear more politically palatable. Conflicts over resources like water are often cloaked in the guise of identity or ideology.

Q. What does the above passage imply?

  1. Resource-based conflicts are always politically motivated.
  2. There are no political solutions to resolve environmental and resource based conflicts.
  3. Environmental issues contribute to resource stresses and political conflict.
  4. Political conflict based on identity or ideology cannot be resolved.

Passage-23

The man who is perpetually hesitating which of the two things he will do first, will do neither. The man who resolves, but suffers his resolution to be changed by the first counter Suggestion of a friend—who fluctuates from opinion to opinion and veers from plan to plan-can never accomplish anything. He will at best be stationary and probably retrograde in all. It is only the man who first consults wisely, then resolves firmly and then executes his purpose with inflexible perseverance, undismayed by those petty difficulties which daunt a weaker spirit—that can advance to eminence in any line.

Q. The keynote that seems to be emerging from the passage is that

  1. we should first consult wisely and then resolve firmly
  2. we should reject suggestions of friends and remain unchanged
  3. we should always remain broad-minded
  4. we should be resolute and achievement-oriented

Passage-24

During the summer in the Arctic Ocean, sea ice ha, been melting earlier and faster, and the winter freeze has been coming later. In the last three decades, the extent of summer ice has declined by about 30 per cent. The lengthening period of summer melt threatens to undermine the whole Arctic food web, atop which stand polar bears.

Q. Which among the following is the most crucial message conveyed by the above passage?

  1. Climate change has caused Arctic summer to be short but temperature to be high.
  2. Polar bears can be shifted to South Pole to ensure their survival.
  3. Without the presence of polar bears, the food chains in Arctic region will disappear.
  4. Climate change poses a threat to the survival of polar bears.

Passage-25

Why do people prefer open defecation and not want toilets or, if they have them, only use them sometimes? Recent research has shown two critical elements: ideas of purity and pollutions, and not wanting pits or septic tanks to fill they have to be emptied. These are the issue that nobody wants to talk about, but if we want to eradicate the practice of open defection, they have to be confronted and dealt properly.

Q. Which among the following is the most crucial message conveyed by the above passage?

  1. The ideas of purity and pollutions are so deep-rooted that they cannot be removed from the minds of the people.
  2. People have to perceive toilet use and pit-emptying as clean and not polluting.
  3. People cannot change their old habits.
  4. People have neither civic sense nor sense of privacy.

Passage-26

In the last two decades, the world's gross domestic product (GDP) has increased 50 percent, whereas inclusive wealth has increased by a mere 6 percent. In recent decades, GDP-driven economic performance, has only harmed inclusive wealth like human capital; and natural capital like forests, land and water. While the world's human capital which stands at 57 percent of total inclusive wealth grew by only 8 percent, the natural which is 23 percent of total inclusive wealth declined by 30 per cent worldwide in the last two decades.

Q. Which of the following is the most crucial inference from the above passage?

  1. More emphasis should be laid on the development of natural capital.
  2. The growth driven by GDP only is neither desirable nor sustainable.
  3. The economic performance of the countries of the world is not satisfactory.
  4. The world needs more human capital under the present circumstances.

Passage-27

By 2020, when the global economy is expected to run short of 56 million young people, India, with its youth surplus of 47 million, could fill the gap. It is in this context that labour reforms are often cited as the way to unlock double-digit growth in India. In 2014, India's labour force was estimated to be about 40 per cent of the population, but 93 per cent of this force was in unorganized sector. Over the last decade, the compound annual growth rate (CAGR) of employment has slowed to 0.5 per cent, with about 14 million jobs created during last year when the labour force increased by about 15 million.

Q. Which of the following is most rational inference from the above passage?

  1. India must control its population growth so as to reduce its unemployment rate.
  2. Labour reforms are required in India to make optimum use of its vast labour force productively
  3. India is poised to achieve the double-digit growth very soon
  4. India is capable of supplying !It, skilled young people to oil, countries.

Passage-28

The very first lesson that should be taught to us when are old enough to understand it, is that complete freedom from the obligation to work is unnatural, and ought to be illegal, as we can escape our share of the burden of work only by throwing it on someone else’s shoulders. Nature ordains that the human race shall perish of famine if it stops working. We cannot escape from this tyranny. The question we have to settle is how much leisure we can afford to allow ourselves.

Q. The main idea of the passage is that

  1. it is essential for human beings to work
  2. there should be a balance between work and leisure
  3. working is a tyranny which we to face
  4. human's understanding of the nature of work is essential

Passage-29

There is no harm in cultivating habits so long as they are not injurious. Indeed, most of us are little more than bundle of habits. Take away our habits and the residuum would hardly be worth bothering about. We could not get on without them. They simplify the mechanism of life. They enable us to do a multitude of things automatically, which, if we had to give fresh and original thought to them each time, would make existence an impossible confusion.

Q. The author suggests that habits

  1. tend to make our lives difficult
  2. add precision to our lives
  3. make it easier for us to live
  4. tend to mechanize our lives


Civil Services Examination 2016


Read the following passages and answer the questions that follow. Your answers to these items should be based on the passage only.

Passage-1

Accountability, or the lack of it, in governance generally, and civil services, in particular, is a major factor underlying the deficiencies in governance and public administration. Designing an effective framework for accountability has been a key element of the reform agenda. A fundamental issue is whether civil services should be accountable to the political executive of the day or to society at large. In other words, how should internal and external accountability be reconciled? Internal accountability is sought to be achieved by internal performance monitoring, official supervision by bodies like the ---Central-Vigilance Commission-and-Comptroller and Auditor—General, and judicial review of executive decisions. Articles 311 and 312 of the Indian Constitution provide job security and safeguards to the civil services, especially the All India Services. The framers of the Constitution had envisaged that provision of these safeguards would result in a civil service that is not totally subservient to the political executive but will have the strength to function in larger public interest. The need to balance internal and external accountability is thus built into the Constitution. The issue is where to draw the line. Over the years, the emphasis seems to have tilted in favour of greater internal accountability of the civil services to the political leaders of the day who in turn are expected to be externally accountable to the society at large through the election process. This system for seeking accountability to Society has not worked out, and has led to several adverse consequences for governance.

Some special measures can be considered for improving accountability in civil services. Provisions of articles 311 and 312 should be reviewed and laws and regulations framed to ensure external accountability of civil services. The proposed Civil Services Bill seeks to address some of these requirements. The respective roles of professional civil services and the political executive should he defined so that professional managerial functions and management of civil services are depoliticized. For this purpose, effective statutory civil service boards should be created at the centre and in the states. Decentralization and devolution of authority to bring government and decision making closer to the people also helps to enhance accountability.

Q. According to the passage, which of the following factor/factors led to the adverse consequences for governance/public administration?

  1. Inability of civil services to strike a balance between internal and external account abilities
  2. Lack of sufficient professional training to the officers of All India Services
  3. Lack of proper service benefits in civil services
  4. Lack of Constitutional provisions to define the respective roles of professional civil services vis-a-vis political executive in this context

Select the correct answer using the code given below:

  1. 1 only
  2. 2 and 3 only
  3. 1 and 4 only
  4. 2, 3 and 4

Q. With reference to the passage, the following assumptions have been made:

  1. Political executive is an obstacle to the accountability of the civil services to the society
  2. In the present framework of Indian polity, the political executive is no longer accountable to the society

Which of these assumptions is/are valid?

  1. 1 only
  2. 2 only
  3. Both 1 and 2
  4. Neither 1 nor 2

Q. Which one of the following is the essential message implied by this passage?

  1. Civil services are not accountable to the society they are serving
  2. Educated and enlightened persons are not taking up political leadership
  3. The framers of the Constitution did not envisage the problems being encountered by the civil services
  4. There is a need and scope for reforms to improve the accountability of civil services

Q. According to the passage, which one of the following is not a means of enhancing internal accountability of civil services?

  1. Better job security and safeguards
  2. Supervision by Central Vigilance Commission
  3. Judicial review of executive decisions
  4. Seeking accountability through enhanced participation by people in decision making process

Passage-2

In general, religious traditions stress our duty to god, or to some universal ethical principle. Our duties to one another derive from these. The religious concept of rights is primarily derived from our relationship to this divinity or principle and the implication it has on our other relationships. This correspondence between rights and duties is critical to any further understanding of justice. But, for justice to be practiced; rights and duties cannot remain formal abstraction. They must be grounded in a community (common unity) bound together by a sense of common union (communion). Even as a personal virtue, this solidarity is essential to the practice and understanding of justice.

Q. With reference to the passage, the following assumptions have been made:

  1. Human relationships are derived from their religious traditions
  2. Human beings can be duty bound only if they believe in god
  3. Religious traditions are essential to practice and understand justice

Which of these assumption(s) is/are valid?

  1. 1 only
  2. 2 and 3 only
  3. 1 and 3 only
  4. 1, 2 and 3

Q. Which one of the following is the crux of this passage?

  1. Our duties to one another derive from our religious traditions
  2. Having relationship to the divine principle is a great virtue
  3. Balance between and duties is crucial to the delivery of justice in a society
  4. Religious concept of rights is primarily derived from our relationship to god

Passage-3

Biomass as fuel for power, heat, and transport has the highest mitigation potential of all renewable sources. It comes from agriculture and forest residues as well as from energy crops. The biggest challenge in using biomass residues is a long-term reliable supply delivered to the power plant at reasonable costs; the key problems are logistical constraints and the costs of fuel collection. Energy crops, if not managed properly, compete with food production and may have undesirable impacts on food prices. Biomass production is also sensitive to the physical impacts of a changing climate.

Projections of the future role of biomass are probably overestimated, given the limits to the sustainable biomass supply, unless breakthrough technologies substantially increase productivity. Climate-energy models project that biomass use could increase nearly four-fold to around 150 — 200 exajoules, almost a quarter of world primary energy in 2050. However the maximum sustainable technical potential of biomass resources (both residues and energy crops) without disruption of food and forest resources ranges from 80 — 170 exajoules a year by 2050, and only part of this is realistically and economically feasible. In addition, some climate models rely on biomass-based carbon capture and storage, an unproven technology, to achieve negative emissions and to buy some time during the first half of the century.

Some liquid biofuels such as corn-based ethanol, mainly for transport, may aggravate rather than ameliorate carbon emissions on a life-cycle basis. Second generation biofuels, based on ligno-cellulosic feedstocks — such as straw, bagasse, grass and wood — hold the promise of sustainable production that is high-yielding and emit low levels of greenhouse gases, but these are still in the R & D stage.

Q. What is/are the present constraint/constraints in using biomass as fuel for power generation?

  1. Lack of sustainable supply of biomass
  2. Biomass production competes with food production
  3. Bio-energy may not always be low carbon on a life-cycle basis

Select the correct answer using the code given below:

  1. 1 and 2 only
  2. 3 only
  3. 2 and 3 only
  4. 1, 2 and 3

Q. Which of the following can lead to food security problem?

  1. Using agricultural and forest residues as feedstock for power generation
  2. Using biomass for carbon capture and storage
  3. Promoting the cultivation of energy crops

Select the correct answer using the code given below:

  1. 1 and 2 only
  2. 3 only
  3. 2 and 3 only
  4. 1, 2 and 3

Q. In the context of using biomass, which of the following is/are the characteristic/characteristics of the sustainable production of biofuel?

  1. Biomass as a fuel for power generation could meet all the primary energy requirements of the world by 2050
  2. Biomass as a fuel for power generation does not necessarily disrupt food and forest resources
  3. Biomass as a fuel for power generation could help in achieving negative emissions, given certain nascent technologies

Select the correct answer using the code given below:

  1. 1 and 2 only
  2. 3 only
  3. 2 and 3 only
  4. 1, 2 and 3

Q. With reference to the passage, following assumptions have been made:

  1. Some climate-energy models suggest that the use of biomass as a fuel for power generation helps in mitigating greenhouse gas emissions
  2. It is not possible to use biomass as a fuel for power generation without disrupting food and forest resources

Which of these assumptions is/are valid?

  1. 1 only
  2. 2 only
  3. Both 1 and 2
  4. Neither 1 nor 2

Passage-4

We are witnessing a dangerous dwindling of biodiversity in our food supply. The green revolution is a mixed blessing. Over time farmers have come to rely heavily on broadly adapted, high yield crops to the exclusion of varieties adapted to the local conditions. Monocropping vast fields with the same genetically uniform seeds helps boost yield and meet immediate hunger needs. Yet high-yield varieties are also genetically weaker crops that require expensive chemical fertilizers and toxic pesticides. In our focus on increasing the amount of food we produce today, we have accidentally put ourselves at risk for food shortages in future.

Q. Which among the following is the most logical and critical inference that can be made from the above passage?

  1. In our agricultural practices, we have become heavily dependent on expensive chemical fertilizers and toxic pesticides only due to green revolution
  2. Monocropping vast fields with high-yield varieties is possible due to green revolution
  3. Monocropping with high-yield varieties is the only way to ensure food security to millions
  4. Green revolution can pose a threat to biodiversity in food supply and food security in the long run

Passage-5

By killing transparency and competition, crony capitalism is harmful to free enterprise, opportunity and economic growth. Crony capitalism, where rich and the influential are alleged to have received land and natural resources and various licences in return for payoffs to venal politicians, is now a major issue to be tackled. One of the greatest dangers to growth of developing economies like India is the middle-income where crony capitalism creates oligarchies that slow down the growth.

Q. Which among the following is the most logical corollary to the above passage?

  1. Launching more welfare schemes and allocating more finances for the current schemes r are urgently needed
  2. Efforts should be made to push up economic growth by other means and provide licences to the poor
  3. Greater transparency in the functioning of the government and promoting the financial inclusion are needed at present
  4. We should concentrate more on developing manufacturing sector than service sector

Passage-6

Climate adaptation may be rendered ineffective if policies are not designed in the context of other development concerns. For instance, a comprehensive strategy that seeks to improve food security in the context of climate change may include a set of coordinated measures related to agricultural extension, crop diversification, integrated water and pest management and agricultural information series. Some of these measures may have to do with climate changes and others with economic development.

Q. What is the most logical and rational inference that can be made from the above passage?

  1. It is difficult to pursue climate adaptation in the developing countries
  2. Improving food security is a far more complex issue than climate adaptation
  3. Every developmental activity is directly or indirectly linked to climate adaptation
  4. Climate adaptation should be examined in tandem with other economic development options

Passage-7

Understanding of the role of biodiversity in the hydrological cycle enables better policy-making. The term biodiversity refers to the variety of plants, animals, microorganisms, and the ecosystems in which they occur. Water and biodiversity are interdependent. In reality, the hydrological cycle decides how biodiversity functions. In turn, vegetation and soil drive the movement of water. Every glass of water we drink has, at least in part, passed through fish, trees, bacteria, soil and other organisms. Passing through these ecosystems, it is cleansed and made fit for consumption. The supply of water is a critical service that the environment provides.

Q. Which among the following is the most critical inference that can be made from the above passage?

  1. Biodiversity sustains the ability of nature to recycle water
  2. We cannot get potable water without the existence of living organisms
  3. Plants, animals and microorganisms continuously interact among themselves
  4. Living organisms could not have come into existence without hydrological cycle

Passage-8

In the last decade, the banking sector has been restructured with a high degree of automation and products that mainly serve middle-class and upper middle-class society. Today there is need for a new agenda for the banking and non-banking financial services that does not exclude the common man

Q. Which one of the following is the message that is essentially implied in the above passage?

  1. Need for more automation and more products of bank
  2. Need for a radical restructuring of our entire public finance system
  3. Need to integrate banking and non-banking institutions
  4. Need to promote financial inclusion

Passage-9

Safe and sustainable sanitation in slums has immeasurable benefits to women and girls in terms of their health, safety, privacy and dimity. However, women do not feature in most of the schemes and policies on urban sanitation. The fact that even now the manual scavenging exists, only goes to show that not enough has been done to promote pour-flush toilets and discontinue the use of dry latrines. A more sustained and rigorous campaign needs to be launched towards the right to sanitation on a very large scale. This should primarily focus on the abolition of manual scavenging.

Q. With reference to the above passage, consider the following statements:

  1. Urban sanitation problems can be fully solved by the abolition of manual scavenging only
  2. There is a need to promote greater awareness on safe sanitation practices in urban areas

Which of the statements given above is/are correct?

  1. 1 only
  2. 2 only
  3. Both I and 2
  4. Neither 1 nor 2

Passage-10

To understand the nature and quantity of Government proper for man, it is necessary to attend to his character. As nature created him for social life, she fitted him for the station she intended. In all cases she made his natural wants greater than his individual powers. No one man is capable, without the aid of society, of supplying his own wants; and those wants, acting upon every individual, impel the whole of them into society.

Q. Which among the following is the most logical and rational inference that can be made from the above passage?

  1. Nature has created a great diversity in human society
  2. Any given human society is always short of its wants
  3. Social life is a specific characteristic of man
  4. Diverse natural wants forced man towards social system

Passage-11

The nature of the legal imperatives in any given state corresponds to the effective demands that state encounters, and that these, in their turn, depend, in a general way, upon the manner in which economic power is distributed in the society which the state controls.

Q. The statement refers to:

  1. the antithesis of Politics and Economics
  2. the interrelationship of Politics and Economics
  3. the predominance of Economics over Politics
  4. the predominance of Politics over Economics

Passage-12

About 15 percent of global greenhouse gas emissions come from agricultural practices. This includes nitrous oxide fertilizers; methane from livestock, rice production, and manure storage; and carbon dioxide (CO2) from burning biomass, but this excludes CO2 emissions from soil management practices, savannah burning and deforestation. Forestry and use, and land-use change account for anotherpercent of greenhouse gas emissions each ear, three quarters of which come from tropical deforestation. The remainder is largely from draining and burning tropical peat land. About the same amount of carbon is stored in the world's peat lands as is stored in the Amazon rainforest.

Q. Which among the following is the most logical and rational inference that can be made from the above passage?

  1. Organic farming should immediately replace mechanized and chemical dependant agricultural practices all over the world
  2. It is imperative for us to modify our land use practices in order to mitigate climate change.
  3. There are no technological solutions to the problem of greenhouse gas emissions
  4. Tropical areas are the chief sites of carbon sequestration

Passage-13

As we look to 2050, when we will need to feed two billion more people, the question of which diet is best has taken on new urgency. The foods we choose to eat in the coming decades will have dramatic ramifications for the planet. Simply put, a diet that revolves around meat and dairy a way of eating that is on the rise throughout the developing world, will take a greater toll on the world's resources than one that revolves around unrefined grains, nuts, fruits and vegetables.

Q. What is the critical message conveyed by the above passage?

  1. Our increasing demand for foods sourced from animals puts a greater burden on our natural resources
  2. Diets based on grains, nuts, fruits and vegetables are best suited for health in developing countries
  3. Human beings change their food habits from time to time irrespective of the health concerns
  4. From a global perspective, we still do not know which type of diet is best for us

Passage-14

All humans digest mother's milk as infants, but until cattle began being domesticated 10,000 years ago, children once weaned no longer needed to digest milk. As a result, they stopped making the enzyme lactase, which breaks down the sugar lactose into simple sugars. After humans began herding cattle, it became tremendously advantageous to digest milk, and lactose tolerance evolved independently among cattle herders in Europe, the Middle East and Africa. Groups not dependant on cattle, such as the Chinese and Thai, remain lactose intolerant.

Q. Which among the following is the most logical assumption that can be made from the above passage?

  1. About 10,000 years ago, the domestication of animals took place in some parts of the world
  2. A permanent change in the food habits of a community can bring about a genetic change in its members
  3. Lactose tolerant people only are capable of getting simple sugars in their bodies
  4. People who are not lactose tolerant cannot digest any dairy product

Passage-15

"The conceptual difficulties in National Income comparisons between underdeveloped and industrialised countries are particularly serious because a part of the national output in various underdeveloped countries is produced without passing through the commercial channels."

Q. In the above statement, the author implies that:

  1. the entire national output produced and consumed in industrialized countries passes through commercial channels
  2. the existence of a non-commercialized sector in different underdeveloped countries renders the national income comparisons over countries difficult
  3. no part of national output should be produced and consumed without passing through commercial channels
  4. a part of the national output being produced and consumed without passing through commercial channels is a sign of underdevelopment

Passage-16

An increase in human-made carbon dioxide in the atmosphere could initiate a chain reaction between plant and microorganisms that would unsettle one of the largest carbon reservoirs on the planet soil In a study, it was found that the soil, which contains twice the amount of carbon present in a plants and Earth's atmosphere combined, could become increasingly volatile people add more carbon dioxide to the atmosphere. This is largely because of increased plant growth. Although a greenhouse gas and a pollutant, carbon dioxide also supports plant growth. As trees and other vegetation flourish in a carbon dioxide-rich future, their roots could stimulate microbial activity in soil that may in turn accelerate the decomposition of soil carbon and its release into the atmosphere as carbon dioxide.

Q. Which among the following is the most logical corollary to the above passage?

  1. Carbon dioxide is essential for the survival of microorganisms and plants
  2. Humans are solely responsible for the release of carbon dioxide into the atmosphere
  3. Microorganisms and soil carbon are mainly responsible for the increased plant growth
  4. Increasing green cover could trigger the release of carbon trapped in soil

Passage-17

Historically, the biggest Challenge to world agriculture has been to achieve a balance between demand for and supply of food. At the level of individual countries, the demand-supply balance can be a critical issue for a closed economy, especially if it is a populous economy and its domestic agriculture is not growing sufficiently enough to ensure food supplies, on an enduring basis; it is not so much and not always, of a constraint for an open, and growing economy, which has adequate exchange surplus to buy food abroad. For the world as a whole, Supply-demand balance is always an inescapable prerequisite for warding off hunger and starvation. However, global availability of adequate supply does not necessarily mean that food would automatically move from countries of surplus to of deficit if the latter lack in purchasing power. The uneven distribution of hunger, starvation, under or malnourishment, etc., at the world-level, thus owes itself to the presence of empty-pocket hungry mouths, overwhelmingly confined to the underdeveloped economies. Inasmuch as 'a two-square meal' is of elemental significance to basic human existence, the issue of worldwide supply` of food has been gaining significance, in recent times, both because the quantum and the composition of demand has been undergoing big changes, and because, in recent years, the capabilities individual countries to generate uninterrupted chain of food supplies have come under strain. Food production, marketing and prices, especially price-affordability by the poor in the developing world, have become global issues that need global thinking and global solutions.

Q. According to the above passage, which of, the following are the fundamental solutions for the world food security problem?

  1. Setting up more agro-based industries
  2. Improving the price affordability by the poor
  3. Regulating the conditions of marketing
  4. Providing food subsidy to one and all

Select the correct answer using the code given below:

  1. 1 and 2
  2. 2 and 3 only
  3. 1, 3 an 4 only
  4. 1, 2, and 4

Q. According to the above passage, the biggest challenge to world agriculture is:

  1. to find sufficient land for agriculture and to expand food processing industries
  2. to eradicate hunger in underdeveloped countries
  3. to achieve a balance between the production of food and non-food items
  4. to achieve a balance between demand for and supply offood

Q. According to the above passage, which of the following helps/help in reducing hunger and starvation in the developing economies?

  1. Balancing demand and supply of food
  2. Increasing imports of food
  3. Increasing purchasing power of the poor
  4. Changing the food consumption patterns and practices

Select the correct answer using the code given below:

  1. 1 only
  2. 2, 3 and 4 only
  3. 1 and 3 only
  4. 1, 2, 3 and 4

Q. The issue of worldwide supply of food has gained importance mainly because of:

  1. overgrowth of the population worldwide
  2. sharp decline in the area of food production
  3. limitation in the capabilities for sustained supply of food

Select the correct answer using the code given below:

  1. 1 and 2 only
  2. 3 only
  3. 2 and 3 only
  4. 1, 2 and 3


Civil Services Examination 2015


Read the following passages and answer the questions that follow. Your answers to these items should be based on the passage only.

Passage-1

Human history abounds m claims and theories confining the right of governing to a few select citizens. Exclusion of the many is justified on the ground that human beings may be rightfully segregated for the good of society and viability of the political process.

Q. Which one of the following statements is least essential as a part of the argument in the above passage?

  1. Man seeks control over external things affecting him.
  2. In society, there are ‘super’ and ‘sub’ human beings.
  3. Exceptions to universal citizen participation are conducive to systemic efficacy.
  4. Governing implies recognition of disparities in individual capacities.

Passage-2

By 2050, the Earth's population will likely have swelled from seven to nine billion people. To fill all those stomachs - while accounting for shifting consumption patterns, climate change, and a finite amount of arable land and potable water - some experts say food production will have to double. How can we make the numbers add up? Experts say higher yielding crop varieties and more efficient farming methods will be crucial. So will waste reduction. Experts urge cities to reclaim nutrients and water from waste streams and preserve farmland. Poor countries, they say, can improve crop storage and packaging and rich nations could cut back on resource-intensive foods like meat.

Q. Which one of the following statements best sums up the above passage?

  1. The population of the world is growing very fast.
  2. Food security is a perennial problem only in developing countries.
  3. The world does not have enough resources to meet the impending food scarcity.
  4. Food security is increasingly a collective challenge.

Passage-3

Many people in India feel that if we cut our defence expenditure on weapon-building, we can create a climate of peace with our neighbours, subsequently reducing the conflict or creating a no-war situation. People who proclaim such ideas are either the victims of war or the propagators of false argument.

Q. With reference to the above passage, which of the following is the most valid assumption?

  1. Building of weapons systems by us has instigated our neighbours to wage wars against us.
  2. The greater spending on weapon-building by us would lessen the possibility of armed conflict with our neighbours.
  3. It is necessary to have state of the art weapons systems for national security.
  4. Many people in India believe that we are wasting our resources on weapon-building.

Passage-4

India accounts for nearly a fifth of the world's child deaths. In terms of numbers, it is the highest in the world - nearly 16 lakhs every year. Of these, more than half die in the first month of life. Officials believe that the reason for this is the absence of steps to propagate basic health practices relating to breast feeding and immunisation. Also the large reproductive population of 2.6 crore remains bereft of care during the critical phases of pregnancy and post-delivery. Added to this is the prevalence of child marriages, anaemia among young women and lack of focus on adolescent sanitation, all of which impact child death rates.

Q. Which is the critical inference that can be made from the above passage?

  1. A lot of Indians are illiterate and hence do not recognize the value of basic health practices.
  2. India has a very huge population and the government alone cannot manage public health services.
  3. Universalization and integration of maternal health and child health services can effectively address the problem.
  4. The nutrition of women in child bearing age does not affect child mortality rate.

Passage-5

Foods travel more than the people who eat them. Grocery stores and supermarkets are loaded with preserved and processed foods. This, however, often leads to environmental threats, such as pollution generated by long distance food transportation and wastage of food during processing and transportation, destruction of rain forests, reduced nutritional content, increased demand for preservation and packaging. Food insecurity also increases as the produce comes from regions that are not feeding their own population properly.

Q. With reference to the above passage, which of the following statements is/are true?

  1. Consuming regionally grown food and not depending on long travelled food is a part of eco-friendly behaviour.
  2. Food processing industry puts a burden on our natural resources.

Select the correct answer using the code given below:

  1. 1 only
  2. 2 only
  3. Both 1 and 2
  4. Neither 1 nor 2

Passage-6

I must say that, beyond occasionally exposing me to laughter, my constitutional shyness has been of no disadvantage whatever. In fact I can see that, on the contrary, it has been all to my advantage. My hesitancy in speech, which was once an annoyance, is now a pleasure. Its greatest benefit has been that it has taught me the economy of words. I have naturally formed the habit of restraining my thoughts. And I can now give myself the certificate that a thoughtless word hardly ever escapes my tongue or pen. I do not recollect ever having had to regret anything in my speech or writing. I have thus been spared many a mishap and waste of time. Experience has taught me that silence is part of the spiritual discipline of a votary of truth. Proneness to exaggerate, to suppress or modify the truth, wittingly or unwittingly, is a natural weakness of man, and silence is necessary in order to surmount it. A man of few words will rarely be thoughtless in his speech; he will measure every word. We find so many people impatient to talk. There is no chairman of a meeting who is not pestered with notes for permission to speak. And whenever the permission is given the speaker generally exceeds the time-limit, asks for more time, and keeps on talking without permission. All this talking can hardly be said to be of any benefit to the world. It is so much waste of time. My shyness has been in reality my shield and buckler. It has allowed me to grow. It has helped me in my discernment of truth.

Q. The author says that a thoughtless word hardly ever escapes his tongue or pen. Which one of the following is not a valid reason for this?

  1. He has no intention to waste his time.
  2. He believes in the economy of words.
  3. He believes in restraining his thoughts.
  4. He has hesitancy in his speech.

Q. The most appropriate reason for the author to be spared many a mishap is that

  1. he hardly utters or writes a thoughtless word.
  2. he is a man of immense patience.
  3. he believes that he is a spiritual person.
  4. he is a votary of truth.

Q. For the author, silence is necessary in order to surmount

  1. constitutional shyness.
  2. hesitancy in speech.
  3. suppression of thoughts.
  4. tendency to overstate.

Passage-7

The richer States have a responsibility to cut down carbon emissions and promote clean energy investments. These are the States that got electricity, grew faster and now have high per capita income, making them capable of sharing India's burden of becoming eco-friendly. Delhi, for example, can help by generating its own clean electricity using solar rooftop panels or even help poor States finance their clean energy projects. It is no secret that State Electricity Boards, which control 95% of the distribution network, are neck-deep in losses. These losses further discourage State utilities from adopting renewable energy as it is more expensive than fossil fuels.

Q. Which among the following is the most logical and rational assumption that can be made from the above passage?

  1. The richer States must lead in the production and adoption of renewable energy.
  2. The poor States always have to depend on rich States for electricity.
  3. The State Electricity Boards can improve their finances by undertaking clean energy projects.
  4. The high economic disparity between the rich and poor States is the major cause of high carbon emissions in India.

Passage-8

Set against a rural backdrop, ‘Stench of kerosene’ is the story of a couple, Guleri and Manak, who have been happily married for several years but do not have a child. Manak’s mother is desperate to have a grandchild to carry on the family name. Hence, she gets Manak remarried in Guleri’s absence. Manak, who acts as a reluctant but passive spectator, is meanwhile, informed by a friend that Guleri, on hearing about her husband’s second marriage, poured kerosene on her clothes and set fire to them. Manak is heartbroken and begins to live as if he were a dead man. When his second wife delivers a son, Manak stares at the child for a long time and blurts out, “Take him away! He stinks of kerosene.”

Q. This is a sensitive issue-based story which tries to sensitize the readers about

  1. Male chauvinism and infidelity
  2. Love and betrayal
  3. Lack of legal safeguards for women
  4. Influence of patriarchal mindset

Passage-9

The ultimate aim of government is not to rule or control by fear, nor to demand obedience, but conversely, to free every man from fear, that he may live in all possible security. In other words, to strengthen his natural right to exist and work without injury to himself or others. The object of government is not to change men from rational beings into beasts or puppets. It should enable them to develop their minds and bodies in security, and to employ their reason unshackled.

Q. Which among the following is the most logical and rational inference that can be made from the above passage?

  1. The true aim of government is to secure the citizens their social and political freedom.
  2. The primary concern of government is to provide absolute social security to all its citizens.
  3. The best government IS the one that allows the citizens to enjoy absolute liberty in all matters of life.
  4. The best government is the one that provides absolute physical security to the people of the country

Passage-10

Our municipal corporations are understaffed. The issue of skills and competencies of the staff poses an even greater challenge. Urban service delivery and infrastructure are complex to plan and execute.

They require a high degree of specialization and professionalism. The current framework within which municipal employees, including senior management, are recruited does not adequately factor in the technical and managerial competencies required. Cadre and recruitment rules only specify the bare minimum in academic qualifications. There is no mention of managerial or technical competencies, or of relevant work experience. This is the case with most municipal corporations. They also suffer from weak organisation design and structure.

Q.Which among the following in the most logical and rational assumption that can be made from the above passage?

  1. The task of providing urban servicing is a complex issue which requires the organizational expansion of municipal bodies all over the country,
  2. Our cities can provide better quality of life if our local government bodies have adequate staff with required skills and competencies.
  3. Lack of skilled staff is due to the absence of institutions which offer the requisite skills in city management.
  4. Our country is not taking advantage of the demographic dividend to manage the problems associated with rapid urbanization

Passage-11

Flamingos in large flock in the wild are social extremely loyal. They perform group mating dances. Parents are very fond of their chicks, gathering them into crèches for protection while both males and females fly off to search for food.

Q. Which among the following is the most logical corollary to the above passage?

  1. Mass nesting in all species of bird is essential to ensure complete survival of their offspring.
  2. Only birds have the capacity to develop social behaviour and thus can do mass nesting to raise their chicks in safety.
  3. Social behaviour in some species of birds increases the odds of' survival in an unsafe world.
  4. All species of birds set up creches for their chicks to teach thorn social behaviour and loyalty

Passage-12

Vast numbers of Indian citizens without bank accounts live in rural areas, are financially and functionally illiterate, and have little experience with technology. A research study was conducted in a particular area in which electronic wage payments in Mahatma Gandhi National Rural Employment Guarantee Scheme (MGNREGS) are meant to go directly to the poor. It was observed that recipients often assume that the village leader needs to mediate the process, as was the case under the previous paper-based system. Among households under this research study area who claimed to have at least one bank account, over a third reported still receiving MGNREGS wages in cash directly from a village leader.

Q. What is the most logical, rational and crucial message that is implied in the above passage?

  1. MGNREGS should be extended only to those who have a bank account.
  2. The paper-based system of payments is more efficient than electronic payment in the present scenario.
  3. The goal of electronic wage payments was not to eliminate mediation by village leaders.
  4. It is essential to provide financial literacy to the rural poor. .

Passage-13

Individuals, groups and leaders who promote human development operate under strong institutional, structural and political constraints that affect policy options. But experience suggests broad principles for shaping an appropriate agenda for human development. One important finding from several decades of human development experience is that focusing exclusively on economic growth is problematic. While we have good knowledge about how to advance health and education, the causes of growth are much less certain and growth is often elusive. Further, an unbalanced emphasis on growth is often associated with negative environmental consequences and adverse distributional effects. The experience of China, with its impressive growth record, reflects these broader concerns and underlines the importance of balanced approaches that emphasize investments in the non-income aspects of human development.

Q. With reference to the above passage, consider the following statements:

  1. In developing countries, a strong institutional framework is the only requirement for human development and policy options
  2. Human development and economic growth are not always positively inter-related.
  3. Focusing only on human development should be the goal of economic growth

Which of the above statements is/are correct?

  1. 1 only
  2. 2 and 3 only
  3. 2 only
  4. 1, 2 and 3

Q. With reference to the above passage, the following assumptions have been made:

  1. Higher economic growth is essential to ensure reduction in economic disparity.
  2. Environmental degradation is sometimes a consequence of economic growth.

Which of the above is/are valid assumption/assumptions?

  1. 1 only
  2. 2 only
  3. Both 1 and 2
  4. Neither 1 nor 2

Passage-14

Climate change is already making many people hungry all over the world, by disrupting crop yields and pushing up prices. And it is not just food but nutrients that are becoming scarcer as the climate changes. It is the poorest communities that will suffer the worst effects of climate change, including increased hunger and malnutrition as crop production and livelihoods are threatened. On the other hand, poverty is a driver of climate change, as desperate communities resort to unsustainable use of resources to meet current needs.

Q. Which among the following is the most logical corollary to the above passage?

  1. Government should allocate more funds to poverty alleviation programmes and increase food subsidies to the poor communities.
  2. Poverty and climate impacts reinforce each other and therefore we have to re-imagine our food systems.
  3. All the countries of the world unite in fighting poverty malnutrition and treat poverty global problem.
  4. We must stop unsustainable agricultural practices control food prices.

Passage-15

The Global Financial Stability Report finds that the share of portfolio investments from advanced economies in the total debt and equity investments in emerging economies has doubled in the past decade to 12 percent. The phenomenon has implications for Indian policy makers as foreign portfolio investments in the debt and equity markets have been on the rise. The phenomenon is also flagged as a threat that could compromise global financial stability in a chain reaction, in the event of United States Federal Reserve's imminent reversal of its "Quantitative Easing" policy.

Q. Which among the following is the most rational and critical inference that can be made from the above passage?

  1. Foreign portfolio investments are not good for emerging economies.
  2. Advanced economies undermine the global financial stability.
  3. India should desist from accepting foreign portfolio investments in the future.
  4. Emerging economies are at a risk of shock from advanced economies.

Passage-16

Open defecation is disastrous when practised in very densely populated areas, where it is impossible to keep away human faeces from crops, wells, food and children's hands. Groundwater is also contaminated by open defecation. Many ingested germs and worms spread diseases. They prevent the body from absorbing calories and nutrients. Nearly one-half of India's children remain malnourished. Lakhs of them die from preventable conditions. Diarrhoea leaves Indians' bodies smaller on average than those of people in some poorer countries where people eat fewer calories. Underweight mothers produce stunted babies prone to sickness who may fail to develop their full cognitive potential. The germs released into environment harm rich and poor alike, even those who use latrines.

Q. Which among the following is the most critical inference that can be made from the above passage?

  1. The Central and State governments in India do not have enough resources to afford a latrine for each household.
  2. Open defecation is the most important public health problem of India.
  3. Open defecation reduces the human capital of India's workforce.
  4. Open defecation is a public health problem in all developing countries.

Passage-17

We generally talk about democracy but when it comes to any particular thing, we prefer a belonging to our caste or community or religion. So long as we have this kind of temptation, our democracy will remain a phoney kind of democracy. We must be in a position to respect a man as a man and to extend opportunities for development to those who deserve them and not to those who happen to belong to our community or race. This fact of favouritism has been responsible for much discontent and ill-will in our country.

Q. Which one of the following statements best sums up the above passage?

  1. Our country has a lot of diversity with its many castes, communities and religions.
  2. True democracy could be established by providing equal opportunities to all.
  3. So far none of us have actually understood the meaning of democracy.
  4. It will never be possible for us to establish truly democratic governance in our country.

Passage-18

The existence/establishment of formal financial institutions that offer safe, reliable and alternative financial instruments is fundamental in mobilising savings. To save, individuals need access to safe and reliable financial institutions, such as banks, and to appropriate financial instruments and reasonable financial incentives. Such access is not always available to all people in developing countries like India and more so, in rural areas. Savings help poor households manage volatility in cash flow, smoothen consumption, and build working capital. Poor households without access to a formal savings mechanism encourage immediate spending temptations.

Q. With reference to the above passage, consider the following statements:

  1. Indian financial institutions do not offer any financial instruments to rural households to mobilise their savings.
  2. Poor households tend to spend their earnings/savings due to lack of access to appropriate financial instruments.

Which of the statements given above is/are correct?

  1. 1 only
  2. 2 only.
  3. Both 1 and 2
  4. Neither 1 nor 2

Q. What is the crucial message conveyed in the passage?

  1. Establish more banks.
  2. Increase the Gross Domestic Product (GDP) growth rate
  3. Increase the interest rate of bank deposits
  4. Promote financial inclusion

Passage-19

Governments may have to take steps which would otherwise be an infringement on the Fundamental Rights of individuals, such as acquiring a person's land against his will, or refusing permission for putting up a building, but the larger public interest for which these are done must be authorized by the people (Parliament). Discretionary powers to the administration can be done away with. It is becoming more and more difficult to keep this power within limits as the government has many number of tasks to perform. Where discretion has to be used, there must be rules and safeguards to prevent misuse of that power. Systems have to be devised which minimise, if not prevent, the abuse of discretionary power. Government work must be conducted within a framework of recognised rules and principles, and decisions should be similar and predictable.

Q. Which among the following is the most logical assumption that can be made from the above passage?

  1. Government should always be given wide discretionary power in all matters of administration.
  2. The supremacy of rules and safeguards should prevail as opposed to the influence of exclusive discretion of authority.
  3. Parliamentary democracy is possible only if the Government has wider discretionary power.
  4. None of the above statements is a logical assumption that can be made from this passage.

Passage-20

India has suffered from persistent high inflation. Increase III administered prices, demand and supply imbalances, imported inflation aggravated by rupee depreciation, and speculation - have combined to keep high inflation going. If there is an element common to all of them, it is that many of them are the outcomes of economic reforms. India's vulnerability to the effects of changes in international prices has increased with trade liberalisation. The effort to reduce subsidies has resulted in a continuous increase in the prices of commodities that are administered.

Q. What is the most logical, rational and crucial message that is implied in the above passage?

  1. Under the present circumstances, India should completely avoid all trade liberalisation policies and all subsidies.
  2. Due to its peculiar socio-economic situation, India is not yet ready for trade liberalisation process.
  3. There is no solution in sight for the problems of continuing poverty and inflation in India in the near future.
  4. Economic reforms can often high inflation economy.

Passage-21

No Right is absolute, exclusive or inviolable. The Right of personal property, similarly, has to be perceived in the larger context of its assumed legitimacy. The Right of personal property should unite the principle of liberty with that of equality, and both with the principle of cooperation.

Q. In the light of the argument in the above passage, which one of the following statements IS the most convincing explanation?

  1. The Right of personal property is a Natural Right duly supported by statutes and scriptures.
  2. Personal property is a theft and an instrument of exploitation. The Right of personal property is therefore violative of economic justice.
  3. The Right of personal property is violative of distributive justice and negates the principle of cooperation.
  4. The comprehensive idea of economic justice demands that the Right of each person to acquisition of property has to be reconciled with that of others.

Passage-22

The conflict between man and State is as old as State history. Although attempts have been made for centuries to bring about a proper adjustment between the competing claims of State and the individual, the solution seems to be' still far off. This is primarily because of the dynamic nature of human society where old values and ideas constantly yield place to new ones. It is obvious that if individuals are allowed to have absolute freedom of speech and action, the result would be chaos, ruin and anarchy.

Q. The author's viewpoint can be best summed up in which of the following statements?

  1. The conflict between the claims of State and individual remains unresolved.
  2. Anarchy and chaos are the obvious results of democratic traditions.
  3. Old values, ideas and traditions persist despite the dynamic nature of human society.
  4. Constitutional guarantee of freedom of speech is not in the interest of society.

Passage-23

Climate change is a complex policy issue with major implications in terms of finance. All actions to address climate change ultimately involve costs. Funding is vital for countries like India to design and implement adaptation and mitigation plans and projects. Lack of funding is a large impediment to implementing adaptation plans. The scale and magnitude of the financial support required by developing countries to enhance their domestic mitigation and adaptation actions are a matter of intense debate in the multilateral negotiations under the United Nations Framework Convention on Climate Change (UNFCCC)' The Convention squarely puts the responsibility for provision of financial support on the developed countries, taking into account their contribution to the stock of greenhouse gases (GHGs) in the atmosphere. Given the magnitude of the task and the funds required, domestic finances are likely to fall short of the current and projected needs of the developing countries. Global funding through the multilateral mechanism of the Convention will enhance their domestic capacity to finance the mitigation efforts.

Q. According to the passage, which of the following is/are a matter of intense debate in the multilateral negotiations under UNFCCC regarding the role of developing countries in climate change?

  1. The scale and size of required financial support.
  2. The crop loss due to climate change in the developing countries.
  3. To enhance the mitigation and adaptation actions in the developing countries.

Select the correct answer using the code given below:

  1. 1 only
  2. 2 and 3 only
  3. 1 and 3 only
  4. 1, 2 and 3

Q. In this passage, the Convention puts the responsibility for the provision of financial support on the developed countries because of

  1. their higher level of per capita incomes.
  2. their large quantum of GDP.
  3. their large contribution to the stock of GHGs in the atmosphere.

Select the correct answer using the code given below:

  1. 1 only
  2. 1 and 2 only
  3. 3 only
  4. 1, 2 and 3

Q. With regards to developing countries, it can be inferred from the passage that climate change is likely to have implications on their

  1. domestic finances.
  2. capacity for multilateral trade.

Select the correct answer using the code given below:

  1. 1 only
  2. 2 only
  3. Both 1 and 2
  4. Neither 1 nor 2

Q. Which one of the following is essentially discussed in the passage?

  1. Conflict between developed and developing countries regarding support for mitigation
  2. Occurrence of climate change due to excessive exploitation of natural resources by the developed countries
  3. Lack of political will on the part of all the countries to implement adaptation plans.
  4. Governance problems of developing countries as a result of climate change


Civil Services Examination 2014


Read the following passages and answer the questions that follow. Your answers to these items should be based on the passage only.

Passage-1

In recent times, India has grown fast not only compared to its own past but also in comparison with other nations. But there cannot be any room for complacency because it is possible for the Indian economy to develop even faster and also to spread the benefits of this growth more widely than has been done thus far. Before going into details of the kinds of micro-structural changes that we need to conceptualize and then proceed to implement, it is worthwhile elaborating on the idea of inclusive growth that constitutes the defining concept behind this Government’s various economic policies and decisions. A nation interested in inclusive growth views the same growth differently depending on whether the gains of the growth are heaped primarily on a small segment or shared widely by the population. The latter is cause for celebration but not the former. In other words, growth must not be treated as an end in itself but as an instrument for spreading prosperity to all. India’s own past experience and the experience of other nations suggests that it is not a sufficient condition. In other words, policies for promoting growth need to be complemented with policies to ensure that more and more people join in the growth process and, further, that there are mechanisms in place to redistribute some of the gains to those who are unable to partake in the market process and, hence, get left behind.

A simple way of giving this idea of inclusive growth a sharper form is to measure a nation’s progress in terms of the progress of its poorest segment, for instance the bottom 20 per cent of the population. One could measure the per capita income of the bottom quintile of the population and also calculate the growth rate of income; and evaluate our economic success in terms of these measures that pertain to the poorest segment. This approach is attractive because it does not ignore growth like some of the older heterodox criteria did. It simply looks at the growth of income of the poorest sections of the population. It also ensures that those who are outside of the bottom quintile do not get ignored. If that were done, then those people would in all likelihood drop down into the bottom quintile and so would automatically become a direct target of our policies. Hence the criterion being suggested here is a statistical summing up of the idea of inclusive growth, which, in turn, leads to two corollaries: to wish that India must strive to achieve high growth and that we must work to ensure that the weakest segments benefit from the growth.

Q. The author’s central focus is on

  1. applauding India’s economic growth not only against its own past performance, but against other nations.
  2. emphasizing the need for economic growth which is the sole determinant of a country’s prosperity.
  3. emphasizing inclusive growth where gains of growth are shared widely by the population.
  4. emphasizing high growth.

Q. The author supports policies which will help

  1. develop economic growth
  2. better distribution of incomes irrespective of rate of growth.
  3. develop economic growth and redistribute economic gains to those getting left behind.
  4. put an emphasis on the development of the poorest segments of society.

Q. Consider the following statements

According to the author, India’s economy has grown but there is no room for complacency as

  1. growth eradicates poverty.
  2. growth has resulted in prosperity for all.

Which of the statements given above is/are correct?

  1. 1 only
  2. 2 only
  3. Both 1 and 2
  4. Neither 1 nor 2

Passage-2

It is easy for the government to control State-owned companies through nods and winks. So what really needs to be done as a first step is to put petrol pricing on a transparent formula – if the price of crude is x and the exchange rate y, then every month or fortnight, the government announces a maximum price of petrol, which anybody can work out from the x and the y. The rule has to be worked out to make sure that the oil-marketing companies can, in general, cover their costs. This will mean that if one company can innovate and cut costs, it will make greater profits. Hence, firms will be more prone to innovate and be efficient under this system.

Once the rule is announced, there should be no interference by the government. If this is done for a while, private companies will re-enter this market. And once a sufficient number of them are in the fray, we can remove the rule-based pricing and leave it truly to the market (subject to, of course, the usual regulations of anti-trust and other competition laws).

Q. Consider the following statements:

According to the passage, an oil company can make greater profits, if a transparent formula for petrol pricing is announced every fortnight or month, by

  1. Promoting its sales.
  2. Undertaking innovation.
  3. Cutting costs.
  4. Selling its equity shares at higher prices.

Which of the statements given above is/are correct?

  1. 1 only
  2. 2 and 3
  3. 3 and 4
  4. 1, 2 and 4

Q. Consider the following statements:

According to the passage, private oil companies re-enter the oil producing market if

  1. A transparent rule-based petrol pricing exists.
  2. There is no government interference in the oil producing market.
  3. Subsidies are given by the government.
  4. Regulations of anti-trust are removed.

Which of the statements given above are correct?

  1. 1 and 2
  2. 2 and 3
  3. 3and 4
  4. 2 and 4

Passage-3

Climate change poses potentially devastating effects of India’s agriculture. While the overall parameters of climate change are increasingly accepted – a 10C average temperature increase over the next 30 years, sea level rise of less than 10 cm in the same period, and regional monsoon variations and corresponding droughts – the impacts in India are likely to be quite site and crop specific. Some crops may respond favourable to the changing conditions, others may not. This emphasizes the need to promote agricultural research and create maximum flexibility in the system to permit adaptations.

The key ingredient for “drought proofing” is the managed recharge of aquifers. To ensure continued yield of important staple crops (e.g. wheat), it may also be necessary to shift the locations where these crops are grown, in response to temperature changes as well as to water availability. The latter will be key factor in making long terms investment decisions.

For example, water runoff from the Himalayas is predicted to increase over the next 30 years as glaciers melt, but then decline substantially thereafter. It will be critical to provide incentives to plan for these large-scale shifts in agro-ecological conditions.

India needs to make long term investment in research and development in agriculture. India islikely to experience changed weather patterns in future.

Q. Consider the following statements:

Climate change may force the shifting of locations of the existing corps due to

  1. Melting of glaciers.
  2. Water availability and temperature suitability at other locations.
  3. Poor productivity of crops.
  4. Wider adaptability of crop plants.

Which of the statements given above are correct?

  1. 1, 2 and 3
  2. 2 and 3 only
  3. 1 and 4 only
  4. 1, 2, 3 and 4

Q. According to the passage, why is it important to promote agricultural research in India?

  1. To predict variations in monsoon patterns and to manage water resources.
  2. To make long term investment decisions for economic growth
  3. To facilitate wider adaptability of crops
  4. To predict drought conditions and to recharge aquifers

Passage-4

It is essential that we mitigate the emissions of greenhouse gases and thus avoid some of the worst impacts of climate change that would take place in coming years and decades. Mitigation would require a major shift in the way we produce and consume energy. A shift away from overwhelming dependence on fossil fuels is now ,long overdue, but unfortunately, technological development has been slow and inadequate largely because government policies have not promoted investments in research and development, myopically as a result of relatively low prices of oil. It is now, therefore, imperative for a country like India treating the opportunity of harnessing renewable energy on a large scale as a national imperative. This country is extremely well endowed with solar, wind and biomass sources of energy. Where we have lagged, unfortunately, is in our ability to develop and to create technological solutions for harnessing these resources.

One particular trajectory for carrying out stringent mitigation of greenhouse gas emissions assessed by the Intergovernmental Panel on Climate Change (IPCC) clearly shows the need for ensuring that global emissions of greenhouse gases peak no later than 2015 and reduce rapidly thereafter. The cost associated with such a trajectory is truly modest and would amount, in the estimation of IPCC, to not more than 3 percent of the global GDP in 2030. In other words, the level of prosperity that the world would have reached without mitigation would at worst be postponed by a few months or a year at the most. This is clearly not a very high price to pay for protecting hundreds of millions of people from the worst risks associated with climate change. Any such effort, however, would require lifestyles to change appropriately also. Mitigation of greenhouse gas emissions is not a mere technological fix, and clearly requires changes in lifestyles and transformation of a country’s economic structure, whereby effective reduction in emissions is brought about, such as through the consumption of much lower quantities of animal protein. The Food and Agriculture Organization (FAO) has determined that the emissions from the livestock sector amount to 18 percent of the total. The reduction of emissions from this source is entirely in the hands of human beings, who have never questioned the impacts that their dietary habits of consuming more and more animal protein are bringing about. Mitigation overall has huge co-benefits, such as lower air pollution and health benefits, higher energy security and greater employment.

Q. According to the passage, which of the following would help in the mitigation of greenhouse gases?

  1. Reducing the consumption of meat
  2. Rapid economic liberalization
  3. Reducing the consumerism
  4. Modern management practices of livestock.

Select the correct answer using the code given below:

  1. 1, 2 and 3
  2. 2, 3 and 4
  3. 1 and 3 only
  4. 2 and 4 only

Q. Why do we continue to depend on the fossil fuels heavily?

  1. Inadequate technological development.
  2. Inadequate funds for research and development.
  3. Inadequate availability of alternative sources of energy.

Select the correct answer using the code given below:

  1. 1 only
  2. 2 and 3 only
  3. 1 and 3 only
  4. 1, 2 and 3

Q. According to the passage, how does the mitigation of greenhouse gases help us?

  1. Reduces expenditure on public health
  2. Reduces dependence on livestock
  3. Reduces energy requirements
  4. Reduces rate of global climate change

Select the correct answer using the code given below:

  1. 1, 2 and 3
  2. 1, 3 and 4
  3. 2, 3 and 4
  4. 1 and 4 only

Q. What is the essential message of the passage?

  1. We continue to depend on fossil fuels heavily
  2. Mitigation of the greenhouse gases is imperative
  3. We must invest in research and development
  4. People must change their lifestyle

Passage-5

The Himalayan ecosystem is highly vulnerable to damage, both due to geological reasons and on account of the stress caused by increased pressure of population, exploitation of natural resources and other related challenges. These aspects may be exacerbated due to the impact of climate change. It is possible that climate change may adversely the Himalayan ecosystem through increased temperature, altered precipitation patterns, episodes of drought and biotic influences. This would not only impact the very sustenance of the indigenous communities in uplands but also the life of downstream dwellers across the country and beyond. Therefore, there is an urgent need for giving special attention to sustain the Himalayan ecosystem. This would require conscious efforts for conserving all the representative systems.

Further, it needs to be emphasized that the endemics with restricted distribution, and most often with specialized habitat requirements, are among the most vulnerable elements. In this respect the Himalayan biodiversity hotspot, with rich endemic diversity, is vulnerable to climate change. The threats include possible loss of genetic resources and species, habitats and concomitantly a decrease in ecosystem services. Therefore, conservation of endemic elements in representative ecosystems/habitats assumes a great significance while drawing conservation plans for the region.

Towards achieving the above, we will have to shift toward contemporary conservation approaches, which include a paradigm of landscape level interconnectivity between protected area systems. The concept advocates a shift from the species-habitat focus to an inclusive focus on expanding the biogeographic range so that natural adjustments to climate change can proceed without being restrictive.

Q. Consider the following statements:

According to the passage, the adverse impact of climate change on an ecosystem can be a

  1. permanent disappearance of some of its flora and fauna.
  2. permanent disappearance of ecosystem itself.

Which of the statements given above is/are correct?

  1. 1 only
  2. 2 only
  3. Both 1 and 2
  4. Neither 1 nor 2

Q. Which one of the following statements best implies the need to shift toward contemporary conservation approach?

  1. Exploitation of natural resources causes a stress on the Himalayan ecosystem.
  2. Climate change alters precipitation patterns, causes episodes of drought and biotic interference.
  3. The rich biodiversity, including endemic diversity, makes the Himalayan region a biodiversity hotspot.
  4. The Himalayan biogeographic region should be enabled to adapt to climate change smoothly.

Q. What is the most important message conveyed by the passage?

  1. Endemism is a characteristic feature of Himalayan region.
  2. Conservation efforts should emphasize on biogeographic ranges rather than on some species or habitats.
  3. Climate change has adverse impact on the Himalayan ecosystem.
  4. Without Himalayan ecosystem, the life of the communities of uplands and down streams will have no sustenance.

Q. With reference to the passage, the following assumptions have been made:

  1. To maintain natural ecosystems, exploitation of natural resources should be completely avoided.
  2. Not only anthropogenic but also natural reasons can adversely affect ecosystems.
  3. Loss of endemic diversity leads to the extinction of ecosystems.

Which of the above assumptions is/are correct?

  1. 1 and 2
  2. 2 only
  3. 2 and 3
  4. 3 only

Passage-6

It is often forgotten that globalization is not only about policies on international economic relationships and transactions, but has equally to do with domestic policies of a nation. Policy changes necessitated by meeting the internationally set conditions (by WTO etc.) of free trade and investment flows obviously affect domestic producers and investors. But the basic philosophy underlying globalization emphasizes absolute freedom to markets to determine prices and production and distribution patterns, and view government interventions as processes that create distortions and bring in inefficiency. Thus, public enterprises have to be privatized through disinvestments and sales; sectors and activities hitherto reserved for the public sector have to be opened to the private sector. This logic extends to the social services like education and health. Any restrictions on the adjustments in workforce by way of retrenchment of workers should also be removed and exit should be made easier by removing any restrictions on closures. Employment and wages should be governed by free play of market forces, as any measure to regulate them can discourage investment and also create inefficiency in production. Above all, in line with the overall philosophy of reduction in the role of the State, fiscal reforms should be undertaken to have generally low levels of taxation and government expenditure should be kept to the minimum to abide by the principle of fiscal prudence. All these are policy actions on the domestic front and are not directly related to the core items of the globalization agenda, namely free international flow of goods and finance.

Q. According to the passage, under the globalization, government interventions are viewed as processes leading to

  1. distortions and inefficiency in the economy.
  2. optimum use of resources.
  3. more profitability to industries.
  4. free play of market forces with regard to industries.

Q. According to the passage, the basic philosophy of globalization is to

  1. give absolute freedom to producers t o determine prices and production.
  2. give freedom to producers to evolve distribution patterns.
  3. give absolute freedom to markets to determine prices, production and employment.
  4. give freedom to producers to import and export.

Q. According to the passage, which of the following is/are necessary for ensuring globalization?

  1. Privatization of public enterprises
  2. Expansionary policy of public expenditure
  3. Free play of market forces t o determine wages and employment.
  4. Privatization of social services like education and health

Select the correct answer using the code given below:

  1. 1 only
  2. 2 and 3 only
  3. 1, 3 and 4
  4. 2, 3 and 4

Q. According to the passage, in the process of globalization the State should have

  1. expanding role.
  2. reducing role.
  3. statutory role.
  4. none of the above roles.

Passage-7

Many nations now place their faith in capitalism and governments choose it as the strategy to create wealth for their people. The spectacular economic growth seen in Brazil, China and India after the liberalization of their economics is proof of its enormous potential and success. However, the global banking crisis and the economic recession have left many bewildered. The debates tend to focus on free market operations and forces, their efficiency and their ability for self correction. Issues of justice, integrity and honesty are rarely elaborated to highlight the failure of the global banking system. The apologists of the system continue to justify the success of capitalism and argue that the recent crisis was a blip.

Their arguments betray an ideological bias with the assumptions that an unregulated market is fair and competent, and that the exercise of private greed will be in the larger public interest.

Few recognize the bidirectional relationship between capitalism and greed; that each reinforces the other. Surely, a more honest conceptualisation of the conflicts of interest among the rich and powerful players who have benefited from the system, their biases and ideology is needed; the focus on the wealth creation should also highlight the resultant gross inequity.

Q. The apologists of the “Free Market System”, according to the passage, believe in

  1. market without control by government authorities.
  2. market without protection by the government
  3. ability of market to self correct
  4. market for free goods and services

Q. With reference to “ideological bias”, the passage implies that

  1. free market is fair but not competent.
  2. free market is not fair but competent.
  3. free market is fair and competent.
  4. free market is neither fair nor biased.

Q. “The exercise of private greed will be in the larger public interest” from the passage

  1. refers to the false ideology of capitalism.
  2. underlies the righteous claims of the free market.
  3. shows the benevolent face of capitalism.
  4. ignores resultant gross inequity.

Which of the statements given above is/are correct?

  1. 1 only
  2. 2 and 3
  3. 1 and 4
  4. 4 only

Passage-8

Net profits are only 22% of their total assets for central public sector undertakings, lower than for the private corporate sector. While the public sector or the State-led entrepreneurship played an important role in triggering India’s industrialization, our evolving development needs, comparatively less-than-satisfactory performance of the public sector enterprises, the maturing of our private sector, a much larger social base now available for expanding entrepreneurship and the growing institutional capabilities to enforce competition policies would suggest that the time has come to review the role of public sector.

What should the portfolio composition of the government be? It should not remain static all times. The airline industry works well as a purely private affair. At the opposite end, rural roads, whose sparse traffic makes tolling unviable, have to be on the balance-sheet of the State. If the government did not own rural roads, they would not exist. Similarly, public health capital in our towns and cities will need to come from the public sector. Equally, preservation and improvement of forest cover will have to be a new priority for the public sector assets.

Take the example of steel. With near-zero tariffs, India is a globally competitive market for the metal. Indian firms export steel into the global market, which demonstrates there is no gap in technology. Indian companies are buying up global steel companies, which shows there is no gap in capital availability. Under these conditions, private ownership works best.

Private ownership is clearly desirable in regulated industries, ranging from finance to infrastructure, where a government agency performs the function of regulation and multiple competing firms are located in the private sector. Here, the simple and clean solution – government as the umpire and the private sector as the players is what works best. In many of these industries, we have a legacy of government ownership, where productivity tends to be lower, fear of bankruptcy is absent, and the risk of asking for money from the tax payer is ever present. There is also the conflict of interest between government as an owner and as the regulator. The formulation and implementation of competition policy will be more vigorous and fair if government companies are out of action.

Q. According to the passage, what is/are the reason/reasons for saying that the time has come to review the role of public sector?

  1. Now public sector has lost its relevance in the industrialization process.
  2. Public sector does not perform satisfactorily.
  3. Entrepreneurship in private sector is expanding.
  4. Effective competition policies are available now.

Which of the statements given above is/are correct in the given context?

  1. 1 and 3 only
  2. 2 only
  3. 2, 3 and 4 only
  4. 1, 2, 3 and 4

Q. According to the passage, rural roads should be in the domain of public sector only. Why?

  1. Rural development work is the domain of government only.
  2. Private sector cannot have monetary gains in this.
  3. Government takes money from tax payers and hence it is the responsibility of government only.
  4. Private sector need not have any social responsibility.

Q. The portfolio composition of the government refers to

  1. Public sector assets quality.
  2. Investment in liquid assets.
  3. Mix of government investment in different industrial sectors.
  4. Buying Return on Investment yielding capital assets.

Q. The author prefers government as the umpire and private sector as players because

  1. Government prescribes norms for a fair play by the price sector.
  2. Government is the ultimate in policy formulation.
  3. Government has no control over private sector players.
  4. None of the above statements is correct in this context.

English Language Comprehension

Passage-1

In front of us was walking a bare-headed old man in tattered clothes. He was driving his beasts. They were all laden with heavy loads of clay from the hills and looked tired. The man carried a long whip which perhaps he himself had made. As he walked down the road he stopped now and then to eat the wild berries that grew on bushes along the uneven road. When he threw away the seeds, the bold birds would fly to peck at them. Sometimes a stray dog watched the procession philosophically and then began to bark. When this happened, my two little sons would stand still holding my hands firmly. A dog can sometimes be dangerous indeed.

Q. The author’s children held his hands firmly because

  1. they were scared of the barking dogs.
  2. they wanted him to pluck berries.
  3. they saw the whip in the old man’s hand.
  4. the road was uneven.

Q. The expression “a stray dog watched the procession philosophically” means that

  1. the dog was restless and ferocious.
  2. the dog stood aloof, looking at the procession with seriousness.
  3. the dog looked at the procession with big, wondering eyes.
  4. the dog stood there with his eyes closed.

Passage-2

Cynthia was a shy girl. She believed that she was plain and untalented. One day her teacher ordered the entire class to show up for audition for the school play. Cynthia nearly died of fright when she was told that she would have to stand on stage in front of the entire class and deliver dialogues. The mere thought of it made her feel sick. But a remarkable transformation occurred during the audition. A thin, shy girl, her knees quaking, her stomach churning in terror, began to stun everyone with her excellent performance. Her bored classmates suddenly stopped their noisy chat to stare at her slender figure on the state. At the end of her audition, the entire room erupted in thunderous applause.

Q. Cynthia was afraid to stand on stage because

  1. she felt her classmates may laugh at her.
  2. her stomach was churning.
  3. she lacked self-confidence.
  4. she did not like school plays.

Q. Cynthia’s classmates were chatting because

  1. it was their turn to act next.
  2. they were bored of the performances.
  3. Cynthia did not act well.
  4. the teacher had no control over them.

Q. Cynthia’s knees were quaking because

  1. she felt nervous and shy.
  2. the teacher scolded her.
  3. she was very thin and weak.
  4. she was afraid of her classmates.

Q. The transformation that occurred during the audition refers to

  1. the nervousness of Cynthia.
  2. the eruption of the entire room in thunderous applause.
  3. the surprise on the faces of her classmates.
  4. the stunning performance of Cynthia.


Civil Services Examination 2013


Read the following passages and answer the questions that follow. Your answers to these items should be based on the passage only.

Passage-1

The subject of democracy has become severely muddled because of the way the rhetoric surrounding it has been used in recent years, There IS, increasingly, an oddly confused dichotomy between those who want to 'impose' democracy on countries In the non-Western world (in these countries' 'own interest', of course) and those who are opposed to such 'imposition' (because of the respect for the countries' 'own ways'). But the entire language of 'imposition', used by both sides, is extraordinarily inappropriate since it makes the implicit assumption that democracy belongs exclusively to the West, taking it to be a quintessentially 'Western' idea which has originated and flourished only in the West.

But the thesis and the pessimism it generates about the possibility of democratic practice in the world would be extremely hard to justify. There were several experiments in local democracy in ancient India. Indeed, in understanding the roots of democracy in the world, we have to take an interest in the history of people participation and public reasoning in different parts of the world. We have to look beyond thinking of democracy only in terms of European and American evolution. We would fail to understand the pervasive demands for participatory living, on which Aristotle spoke with far-reaching insight, if we take democracy to be a kind of a specialized cultural product of the West.

It cannot, of course, be doubted that the institutional structure of the contemporary practice of democracy is largely the product of European and American experience over the last few centuries. This is extremely important to recognize since these developments in institutional formats were immensely innovative and ultimately effective. There can be little doubt that there is a major 'Western' achievement here.

Q. Which of the following is closest to the view of democracy as mentioned in the above passage?

  1. The subject of democracy is a muddle due to a desire to portray it as a Western concept, 'alien' to non-Western countries.
  2. The language or imposition of democracy is inappropriate. There is, however, a need to consider this concept in the backdrop of culture of 'own ways' of non-Western society.
  3. While democracy is not essentially a Western idea belonging exclusively to the West, the institutional structure of current democratic practices has been their contribution.
  4. None of the statements (a), (b) and (c) given above is correct.

Q. With reference to the passage, the following assumptions have been made:

  1. Many of the non-Western countries are unable to have democracy because they take democracy to be a specialized cultural product of the West.
  2. Western countries are always trying to impose democracy on non-Western countries.

Which of the above is/are valid assumption/assumptions?

  1. 1 only
  2. 2 only
  3. Both 1 and 2
  4. Neither 1 nor 2

Passage-2

Corporate governance is based on principles such as conducting the business with all integrity and fairness, being transparent with regard to all transactions, making all the necessary disclosures and decisions, complying with all the laws of the land, accountability and responsibility towards the stakeholders and commitment to conducting business in an ethical manner. Another point which is highlighted on corporate governance is the need for those in control to be able to distinguish between what are personal and corporate funds while managing a company.

Fundamentally, there is a level of confidence that is associated with a company that is known to have good corporate governance. The presence of an active group of independent directors on the board contributes a great deal towards ensuring confidence m the market. Corporate governance is known to be one of the criteria that foreign institutional investors are increasingly depending on when deciding on which companies to invest in. It is also known to have a positive influence on the share price of the company. Having a clean image on the corporate governance front could also make it easier for companies to source capital at more reasonable costs. Unfortunately, corporate governance often becomes the centre of discussion only after the exposure of a large scam.

Q. According to the passage, which of the following should be the practice/practices in good corporate governance?

  1. Companies should always comply with labour and tax laws of the land.
  2. Every company in the country should have a government 'representative as one of the independent directors on the board to ensure transparency.
  3. The manager of a company should never invest his personal, funds in the company.

Select the correct answer using the codes given below:

  1. 1 only
  2. 2 and 3 only
  3. 1 and 3 only
  4. 1, 2 and 3

Q. According to the passage, which of the following is/are the major benefit/benefits of good corporate governance?

  1. Good corporate governance leads to increase in share price of the company.
  2. A company with good corporate governance always increases its business turnover rapidly.
  3. Good corporate governance is the main criterion for foreign institutional investors when they decide to buy a company.

Select the correct answer using the codes given below:

  1. 1 only
  2. 2 and 3 only
  3. 1 and 3 only
  4. 1, 2 and 3

Passage-3

Malnutrition most commonly occurs between the ages of six months and two years. This happens despite the child's food requirements being less than that of an older child. Malnutrition is often attributed to poverty, but it has been found that even in households where adults eat adequate quantities of food, more than 50 per cent of children-under-five do not consume enough food. The child's dependence on someone else to feed him/her is primarily responsible for the malnutrition. Very often the mother is working and the responsibility of feeding the young child is left to an older sibling. It is therefore crucial to increase awareness regarding the child's food needs and how to satisfy them.

Q. According to the passage, malnutrition in children can be reduced

  1. if the children have regular intake of food.
  2. after they cross the age of five.
  3. if the food needs of younger children are known.
  4. if the responsibility of feeding younger children is given to adults.

Q. According to the author, poverty is not the main cause of malnutrition, but the fact that

  1. taking care of younger ones is not a priority for working mothers.
  2. awareness of nutritional needs is not propagated by the Public Health authorities.

Select the correct answer using the code given below:

  1. 1 only
  2. 2 only
  3. Both 1 and 2
  4. Neither 1 nor 2

Passage-4

A number of empirical studies find that farmers are risk-averse, though only moderately in many cases. There is also evidence to show that farmers' risk aversion results in cropping patterns and input use designed to reduce risk rather than to maximize income. Farmers adopt a number of strategies to manage and cope with agricultural risks. These include practices like crop and field diversification, non-farm employment, storage of stocks and strategic migration of family members. There are also institutions ranging from share tenancy to kinship, extended family and informal credit agencies. One major obstacle to risk sharing by farmers is that the same type of risks can affect a large number of farmers in the region. Empirical studies show that the traditional methods are not adequate. Hence there is a need for policy interventions, especially measures that cut across geographical regions.

Policies may aim at tackling agricultural risks directly or indirectly. Examples of risk-specific policies are crop insurance, price stabilization and the development of varieties resistant to pests and diseases. Policies which affect risk indirectly are irrigation, subsidized credit and access to information. No single risk-specific policy is sufficient to reduce risk and is without side-effects, whereas policies not specific to risk influence the general situation and affect risks only indirectly. Crop insurance, as a policy measure to tackle agricultural risk directly, deserves careful consideration in the Indian context and in many other developing countries because the majority of farmers depend on rain-fed agriculture and in many areas yield variability is the predominant cause of their income instability.

Q. The need for policy intervention to mitigate risks in agriculture is because

  1. farmers are extremely risk-averse.
  2. farmers do not know how to mitigate risks.
  3. the methods adopted by farmers and existing risk sharing institutions are not adequate.
  4. majority of farmers depend on rain-fed agriculture.

Q. Which of the following observations emerges from the above passage?

  1. One can identify a single policy that can reduce risk without any side-effect.
  2. No single risk-specific policy is sufficient to reduce agricultural risk.
  3. Policies which affect risk indirectly can eliminate it.
  4. Government's policy intervention can mitigate agricultural risk completely.

Passage-5

Financial markets in India have acquired greater depth and liquidity over the years. Steady reforms since 1991 have led to growing linkages and integration of the Indian economy and its financial system with the global economy. Weak global economic prospects and continuing uncertainties the in international financial markets therefore, have had their impact on the emerging market economies. Sovereign risk concerns, particularly in the Euro area, affected financial markets for the greater part of the year, with the contagion of Greece's sovereign debt problem spreading to India and other economies by way of higher-than-normal levels of volatility.

The funding constraints in international financial markets could impact both the availability and cost of foreign funding for banks and corporates. Since the Indian financial system is bank dominated, banks' ability to withstand stress is critical to overall financial stability. Indian banks, however, remain robust, notwithstanding a decline in capital to risk-weighted assets ratio and a rise in non-performing asset levels in the recent past. Capital adequacy levels remain above the regulatory requirements. The financial market infrastructure continues to function without any major disruption. With further globalization, consolidation, deregulation, and diversification of the financial system, the banking business may become more complex and riskier. Issues like risk and liquidity management and enhancing skill therefore assume greater significance.

Q. According to the passage, the financial markets in the emerging market economies including India had the adverse impact in recent years due to

  1. weak global economic prospects.
  2. uncertainties in the international financial markets.
  3. sovereign risk concerns in the Euro area.
  4. bad monsoons and the resultant crop loss.

Select the correct answer using the code given below:

  1. 1 and 2 only
  2. 1, 2 and 3
  3. 2 and 3 only
  4. 2, 3 and 4

Q. The Indian financial markets are affected by global changes mainly due to the

  1. increased inflow of remittances from abroad.
  2. enormous increase in the foreign exchange reserves.
  3. growing global linkages and integration of the Indian financial markets.
  4. contagion of Greece's sovereign debt problem.

Q. According to the passage, in the Indian financial system, banks' ability to with stand stress is critical to ensure overall financial stability because Indian financial system is

  1. controlled by the Government of India.
  2. less integrated with banks.
  3. controlled by Reserve Bank of India.
  4. dominated by banks.

Q. Risk and liquidity management assumes more importance in the Indian banking system in future due to

  1. further globalization.
  2. more consolidation and deregulation of the financial system.
  3. further diversification of the financial system.
  4. more financial inclusion in the economy.

Select the correct answer using the code given below:

  1. 1, 2 and 3
  2. 2, 3 and 4
  3. 1 and 2 only
  4. 3 and 4 only

Passage-6

Crude mineral oil comes out of the earth as a thick brown or black liquid with a strong smell. It is a complex mixture of many different substances, each with its own individual qualities. Most of them are combinations of hydrogen and carbon in varying proportions. Such hydrocarbons are also found in other forms such as bitumen, asphalt and natural gas. Mineral oil originates from the carcasses of tiny animals and from plants that live in the sea. Over millions of years, these dead creatures form large deposits under the sea-bed; and ocean currents cover them with a blanket of sand and silt. As this mineral hardens, it becomes sedimentary rock and effectively shuts out the oxygen, so preventing the complete decomposition of the marine deposits underneath. The layers of sedimentary rock become thicker and heavier. Their pressure produces heat, which transforms the tiny carcasses into crude oil in a process that is still going on today.

Q. Mineral oil deposits under the sea do not get completely decomposed because they

  1. are constantly washed by the ocean currents.
  2. become rock and prevent oxygen from entering them.
  3. contain a mixture 'of hydrogen and carbon.
  4. are carcasses of organisms lying in saline conditions.

Q. Sedimentary rock leads to the formation of oil deposits because

  1. there are no saline conditions below it.
  2. it allows some dissolved oxygen to enter the dead organic matter below it.
  3. weight of overlying sediment layers causes the production of heat.
  4. it contains the substances that catalyze the chemical reactions required to change dead organisms into oil.

Passage-7

Ecological research over the last quarter of the century has established the deleterious effects of habitat fragmentation due to mining, highways and such other intrusions on forests. When a large block of forests gets fragmented into smaller bits, the edges of all these bits come into contact with human activities resulting in the degradation of the entire forests. Continuity of forested landscapes and corridors gets disrupted 'affecting several extinction-prone species of wildlife. Habitat fragmentation is therefore considered as the most serious threat to biodiversity conservation. Ad hoc grants of forest lands to mining companies coupled with rampant, illegal mining is aggravating this threat.

Q. What is the central focus of this passage?

  1. Illegal mining in forests
  2. Extinction of wildlife
  3. Conservation of nature
  4. Disruption of habitat

Q. What is the purpose of maintaining the continuity of forested landscapes and corridors?

  1. Preservation of biodiversity.
  2. Management of mineral resources.
  3. Grant of forest lands for human activities.

Select the correct answer using the codes given below:

  1. 1 only
  2. 1 and 2
  3. 2 and 3
  4. 1, 2 and 3

Passage-8

The law in many parts of the world increasingly restricts the discharge of agricultural slurry into watercourses. The simplest and often the most economically sound practice returns the material to the land as semisolid manure or as sprayed slurry. This dilutes its concentration in the environment to what might have occurred III a more primitive and sustainable type of agriculture and converts pollutant into fertilizer. Soil microorganisms decompose the organic components of sewage and slurry and most of the mineral nutrients become available to be absorbed again by the vegetation.

The excess input of nutrients, both nitrogen and phosphorus - based, from agricultural runoff (and human sewage) has caused many 'healthy' oligotrophic lakes (low nutrient concentrations, low plant productivity with abundant water weeds, and clear water) to change to eutrophic condition where high nutrient inputs lead to high phytoplankton productivity (sometimes dominated by bloom-forming toxic species). This makes the water turbid, eliminates large plants and, in the worst situations, leads to anoxia and fish kills; so called cultural eutrophication. Thus, important ecosystem services arc lost, including the provisioning service of wild-caught fish and the cultural services associated with recreation.

The process of cultural eutrophication of lakes has been understood for some time. But only recently did scientists notice huge 'dead zones' in the oceans near river outlets, particularly those draining large catchment areas such as the Mississippi in North America and the Yangtze in China. The nutrient-enriched water flows through streams, rivers and lakes, and eventually to the estuary and ocean where the ecological impact may be huge, killing virtually all invertebrates and fish in areas up to 70,000 km2 in extent. More than 150 sea areas worldwide are now regularly starved of oxygen as a result of decomposition of algal blooms, fuelled particularly by nitrogen from agricultural runoff of fertilizers and sewage from large cities. Oceanic dead zones are typically associated with industrialized nations and usually lie off'- 'countries that subsidize their agriculture, encouraging farmers to increase productivity and use more fertilizer.

Q. According to the passage, why should the discharge of agricultural slurry into watercourses be restricted?

  1. Losing nutrients in this way is not a good practice economically.
  2. Watercourses do not contain the microorganisms that can decompose organic components of agricultural slurry.
  3. The discharge may lead to the eutrophication of water bodies.

Select the correct answer using the codes given below:

  1. 1 only
  2. 2 and 3 only
  3. 1 and 3 only
  4. 1, 2 and 3

Q. The passage refers to the conversion of "pollutant to fertilizer". What is pollutant and what is fertilizer in this context?

  1. Decomposed organic component of slurry is pollutant and microorganisms in soil constitute fertilizer.
  2. Discharged agricultural slurry is pollutant and decomposed slurry in soil is fertilizer.
  3. Sprayed slurry is pollutant and watercourse is fertilizer.
  4. None of the above expressions is correct in this context.

Q. According to the passage, what are the effects of indiscriminate use of fertilizers?

  1. Addition of pollutants to the soil and water.
  2. Destruction decomposer of microorganisms in soil.
  3. Nutrient enrichment of water bodies.
  4. Creation of algal blooms.

Select the correct answer from the codes given below:

  1. 1, 2 and 3 only
  2. 1, 3 and 4 only
  3. 2 arid 4 only
  4. 1, 2, 3 and 4

Q. What is/are the characteristics of a water body with cultural eutrophication?

  1. Loss of ecosystem services
  2. Loss of flora and fauna
  3. Loss of mineral nutrients

Select the correct answer using the code given below:

  1. 1 only
  2. 1 and 2 only
  3. 2 and 3 only
  4. 1, 2 and 3

Q. What is the central theme of this passage?

  1. Appropriate legislation is essential to protect the environment.
  2. Modern agriculture is responsible for the destruction of environment.
  3. Improper waste disposal from agriculture can destroy the aquatic ecosystems.
  4. Use of chemical fertilizers is undesirable in agriculture.

Passage-9

The miseries of the world cannot be cured by physical help only. Until man's nature changes, his physical needs will always arise, and miseries will always be felt, and no amount of physical help will remove them completely. The only solution of the problem is to make mankind pure. Ignorance is the mother of evil and of all the misery we sec. Let men have light, let them be pure and spiritually strong and educated; then alone will misery cease in the world. We may convert every house in the country into a charitable asylum, we may fill the land with hospitals, but human misery will continue until man's character changes.

Q. According to the passage, which of the following statements is most likely to be true as the reason for man's miseries?

  1. The poor economic and social conditions prevailing in society.
  2. The refusal on the part of man to change his character.
  3. The absence of physical and material help from his society.
  4. Ever increasing physical' needs due to changing social structure.

Q. With reference to the passage, the following assumptions have been made:

  1. The author gives primary importance to physical and material help in eradicating human misery.
  2. Charitable homes, hospitals, etc. can remove human misery to a great extent.

Which of the assumptions is/are valid?

  1. 1 only
  2. 2 only
  3. Both 1 and 2
  4. Neither 1 nor 2


The following eight items are based on three passages in English to test the comprehension of English language. Read each passage and answer the items that follow.

English Language Comprehension

Passage - 1

Seven-year-old Jim came home from the park without his new bicycle. "An old man and a little boy borrowed it," he explained. "They are going to bring it back at four o'clock." His parents were upset that he had given his expensive new bicycle, but were secretly proud of his kindness and faith. Came four o'clock, no bicycle. The parents were anxious. But at 4:30, the door bell rang, and there stood a happy man and a boy, with the bicycle and a box of chocolates. Jim suddenly disappeared into his bedroom, and then came running out. "All right," he said, after examining the bicycle. ''You can have your watch back!"

Q. When Jim came home without his bicycle, his parents

  1. were angry with him.
  2. were worried.
  3. did not feel concerned.
  4. were eager to meet the old man and the little boy.

Q. Jim returned the watch to the old man and the little boy because

  1. they gave him chocolates.
  2. his father was proud of him.
  3. he was satisfied with the condition of his bicycle.
  4. they were late only by 30 minutes.

Passage - 2

It was already late when we set out for the next town, which according to the map was about fifteen kilometres away on the other side of the hills. There we felt that we would find a bed for the night. Darkness fell soon after we left the village, but luckily we met no one as we drove swiftly along the narrow winding road that led to the hills. As we climbed higher, it became colder and rain began to fall, making it difficult at times to see the road. I asked John, my companion, to drive more slowly. After we had travelled for about twenty kilometres, there was still no sign of the town which was marked on the map. We were beginning to get worried. Then without warning, the car stopped and we found we had run out of petrol.

Q. The author asked John to drive more slowly because

  1. the road led to the hills.
  2. John was an inexperienced driver.
  3. the road was not clearly visible.
  4. they were in wilderness.

Q. The travellers set out for the town although it was getting dark because

  1. they were in a hurry.
  2. the next town was a short distance away and was a hill-resort.
  3. they were in wilderness.
  4. the next town was a short distance away and promised a good rest for the night.

Q. The travellers were worried after twenty kilometres because

  1. it was a lonely countryside.
  2. they probably feared of having lost their way.
  3. the rain began to fall
  4. it was getting colder as they drove.

Passage - 3

A stout old lady was walking with her basket down the middle of a street in Petrograd to the great confusion of the traffic and no small peril to herself. It was pointed out to her that the pavement was the place for foot-passengers, but she replied, "I m going to walk where I like. We've got liberty now." It did not occur to the dear lady that if liberty entitled the foot-passenger to walk down the middle of the road it also entitled the taxi-driver to drive on the pavement, and that the end of such liberty would be universal chaos. Everything would be getting in everybody else's way and nobody would get anywhere. Individual liberty would have become social anarchy.

Q. It was pointed out to the lady that she should walk on the pavement because she was

  1. a pedestrian.
  2. carrying a basket.
  3. stout.
  4. an old lady.

Q. The lady refused to move from the middle of the street because

  1. she was not afraid of being killed.
  2. she felt that she is entitled to do whatever she liked.
  3. she did not like walking on the pavement.
  4. she was confused.

Q. The old lady failed to realise that

  1. she was not really free.
  2. her liberty was not unlimited.
  3. she was an old person.
  4. roads are made for motor vehicles only.


Civil Services Examination 2012


Read the following passages and answer the questions that follow. Your answers to these items should be based on the passage only.

Passage-1

Education, without a doubt, has an important functional, instrumental and utilitarian dimension. This is revealed when one asks questions such as 'what is the purpose of education?'. The Solutions, too often, are 'to acquire qualifications for employment/upward mobility', 'wider/higher (in terms of income) opportunities', and 'to meet the needs for trained human power in diverse field for national development'. But in its deepest sense education is not instrumentalist. That is to say, it is not to be justified outside of itself because it leads to the acquisition of formal skills or of certain desired psychological – social attributes. It must be respected in itself. Education is thus not a commodity to be acquired or possessed and then used, but a process of inestimable importance to individuals and society, although it can and does have enormous use value. Education then, is a process of expansion and conversion, not in the sense of conversion turning students into doctors or engineers, but the widening and turning out of the mind – the creation, sustenance and development of self-critical awareness and independence of thought. It is an inner process of moral-intellectual development.

Q. What do you understand by the 'instrumentalist' view of education?

  1. Education is functional and utilitarian in its purposes.
  2. Education is meant to fulfill human needs
  3. The purpose of education is to train the human intellect
  4. Education is meant to achieve moral development

Q. According to the passage, education must be respected in itself because

  1. It helps to acquire qualifications for employment.
  2. It helps in upward mobility and acquiring social status.
  3. It is an inner process of moral and intellectual development
  4. All the (a), (b) and (c) given above are correct in this context.

Q. Education is a process in which

  1. Students are converted into trained professionals.
  2. Opportunities for higher income are generated
  3. individuals develop self-critical awareness and independence of thought
  4. qualifications for upward mobility are acquired

Passage-2

Chemical pesticides lose their role in sustainable agriculture if the pests evolve resistance. The evolution of the pesticide resistance is simply natural selection in action. It is almost certain to occur when vast numbers of a genetically variable population are killed. One or a few individuals may be unusually resistant (perhaps because they possess an enzyme that can detoxify the pesticide). If the pesticide is applied repeatedly, each successive generation of the pest will contain a larger proportion of resistant individuals. Pests typically have a high intrinsic rate of reproduction, and so a few individuals in one generation may give rise to hundreds or thousands in the next, and resistance spreads very rapidly in a population.

This problem was often ignored in the past, even though the first case of DDT (dichlorodiphenyltrichloroethane) resistance was reported as early as 1946. There is an exponential increase in the numbers of invertebrates that have evolved resistance and in the number pesticides against which resistance has evolved. Resistance has been recorded in every family of arthropod pests (including dipterans such as mosquitoes and house flies, as well as beetles, moths, wasps, fleas, lice and mites) as well as in weeds and plant pathogens. Take the Alabama leaf worm, a moth pest of cotton, as an example. It has developed resistance in one or more regions of the world to aldrin, DDT, dieldrin, endrin, lindane and toxaphene.

If chemical pesticides brought nothing but problems, - if their use was intrinsically and acutely unsustainable – then they would already have fallen out of widespread use. This has not happened. Instead, their rate of production has increased rapidly. The ratio of cost to benefit for the individual agricultural producer has remained in favour of pesticide use. In the USA, insecticides have been estimated to benefit the agricultural products to the tune of around $5 for every $1 spent.

Moreover, in many poorer countries, the prospect of imminent mass starvation, or of an epidemic disease, are so frightening that the social and health costs of using pesticides have to be ignored. In general the use of pesticides is justified by objective measures such as 'lives saved', 'economic efficiency of food production' and 'total food produced'. In these very fundamental senses, their use may be described as sustainable. In practice, sustainability depends on continually developing new pesticides that keep at least one step ahead of the pests – pesticides that are less persistent, biodegradable and more accurately targeted at the pests.

Q. “The evolution of pesticide resistance is natural selection in action.” What does it actually imply?

  1. It is very natural for many organisms to have pesticide resistance.
  2. Pesticide resistance among organisms is a universal phenomenon.
  3. Some individuals in any given population show resistance after the application of pesticides
  4. None of the statements a), b) and c) given above is correct.

Q. With reference to the passage, consider the following statements:

  1. Use of chemical pesticides has become imperative in all the poor countries of the world.
  2. Chemical pesticides should not have any role in sustainable agriculture
  3. One pest can develop resistance to many pesticides

Which of the statements given above is/are correct?

  1. 1 and 2 only
  2. 3 only
  3. 1 and 3 only
  4. 1, 2 and 3

Q. Though the problems associated with the use of chemical pesticides is known for a long time, their widespread use has not waned. Why?

  1. Alternatives to chemical pesticides do not exist at all.
  2. New pesticides are not invented at all.
  3. Pesticides are biodegradable.
  4. None of the statements a), b) and c) given above is correct.

Q. How do pesticides act as agents for the selection of resistant individuals in any pest population?

  1. It is possible that in a pest population the individuals will behave differently due to their genetic makeup.
  2. Pests do possess the ability to detoxify the pesticides.
  3. Evolution of pesticide resistance is equally distributed in pest population.

Which of the statements given above is/are correct?

  1. 1 only
  2. 1 and 2 only
  3. 3 only
  4. 1, 2 and 3

Q. Why is the use of chemical pesticides generally justified by giving the examples of poor and developing countries?

  1. Developed countries can afford to do away with use of pesticides by adapting to organic farming, but it is imperative for poor and developing countries to use chemical pesticides.
  2. In poor and developing countries, the pesticide addresses the problem of epidemic diseases of crops and eases the food problem.
  3. The social and health costs of pesticide use are generally ignored in poor and developing countries.

Which of the statements given above is/are correct?

  1. 1 only
  2. 1 and 2 only
  3. 2 only
  4. 1, 2 and 3

Q. What does the passage imply?

  1. Alternative options to chemical pesticides should be promoted.
  2. Too much use of chemicals is not good for the ecosystem.
  3. There is no scope for the improvement of pesticides and making their use sustainable
  4. Both the statements (a) and (b) above are correct.

Passage-3

Today's developing economies use much less energy per capita than developed countries such as the United State did at similar incomes, showing the potential for lower-carbon growth. Adaptation and mitigation need to be integrated into a climate-smart development strategy that increases resilience, reduces the threat of further global warming, and improves development outcomes. Adaptation and mitigation measures can advance development, and prosperity can raise incomes and foster better institutions. A healthier population living in better – built houses and with access to bank loans and social security is better equipped to deal with a changing climate and its consequences. Advancing robust, resilient development policies that promote adaptation is needed today because changes is the climate, already begun, will increase even in the short term.

The spread of economic prosperity has always been intertwined with adaptation to changing ecological conditions. But as growth has altered the environment and as environmental change has accelerated, sustaining growth and adaptability demands greater capacity to understand our environment, generate new adaptive technologies and practices, and diffuse them widely. As economic historians have explained, much of humankind's creative potential has directed at adapting to the changing world. But adaptation cannot cope with all the impacts related to climate change, especially as larger changes unfold in the long term.

Countries cannot grow out of harm's way fast enough to match the changing climate. And some growth strategies, whether driven by the government or the market, can also add to vulnerability – particularly if they over exploit natural resources. Under the Soviet development plan, irrigated cotton cultivation expanded in water-stressed Central Asia and led to the near disappearance of the Aral Sea, threatening the livelihoods of fishermen, herders and farmers. And clearing mangroves- the natural coastal buffers against storm surges – to make way for intensive farming or housing development, increases the physical vulnerability of coastal settlements, whether in Guinea or in Louisiana.

Q. Which of the following conditions of growth can add to vulnerability?

  1. When the growth occurs due to excessive exploitation of mineral resources and forests
  2. When the growth brings about a change in humankind's creative potential.
  3. When the growth is envisaged only for providing houses and social security to the people.
  4. When the growth occurs due to emphasis on farming only.

Select the correct answer using the codes given below:

  1. 1 only
  2. 2, 3 and 4 only
  3. 1 and 4 only
  4. 1, 2, 3 and 4

Q. What does low-carbon growth imply in the present context?

  1. More emphasis on the use of renewable sources of energy.
  2. Less emphasis on manufacturing sector and more emphasis on agricultural sector.
  3. Switching over from monoculture practices to mixed farming
  4. Less demand for goods and services.

Select the correct answer using the codes given below:

  1. 1 only
  2. 2, 3 and 4 only
  3. 1 and 4 only
  4. None of the above implies low-carbon growth

Q. Which of the following conditions is/are necessary for the sustainable economic growth?

  1. Spreading of economic prosperity more.
  2. Popularizing/spreading of adaptive technologies widely
  3. Investing on research in adaptation and mitigation technologies.

Select the correct answer using the codes given below:

  1. 1 only
  2. 2 and 3 only
  3. 1 and 3 only
  4. 1, 2, and 3

Q. Which of the following inferences can be made from the passage?

  1. Rainfed crops should not be cultivated in irrigated areas
  2. Farming under water-deficient areas should not be a part of development strategy.

Select the correct answer using the codes given below:

  1. 1 only
  2. 2 only
  3. Both 1 and 2
  4. Neither 1 nor 2

Q. Consider the following assumptions:

  1. Sustainable economic growth demands the use of creative potential of man.
  2. Intensive agriculture can lead to ecological backlash.
  3. Spread of the economic prosperity can adversely affect the ecology and environment

With reference to the passage, which of the above assumptions is/are valid?

  1. 1 only
  2. 2 and 3 only
  3. 1 and 3 only
  4. 1, 2 and 3

Q. Which one of the following statements constitutes the central theme of this passage?

  1. Countries with greater economic prosperity are better equipped to deal with the consequences of climate change.
  2. Adaptation and mitigation should be integrated with development strategies
  3. Rapid economic growth should not be pursued by both developed and developing economies.
  4. Some countries resort to over exploitation of natural resources for the sake of rapid development.

Passage-4

Invasions of exotic species into new geographical areas sometimes occur naturally without human agency. However, human actions have increased this trickle to a flood. Human caused introductions may occur either accidently or as a consequence of human transport, or intentionally but illegally to serve some private purpose or legitimately to procure some hoped-for public benefit by bringing a pest under control, producing new agricultural products or providing novel recreational opportunities. Many introduced species are assimilated into communities without much obvious effect. However some have been responsible for dramatic changes to native species and native communities. For example, the accidental introduction of the brown tree snake, Boiga irregularis into Guam, an island in the pacific, has through nest predation reduced 10 endemic forest bird species to the point of extinction.

One of the major reasons for the world’s great biodiversity is the occurrence of centres of endemism so that similar habitats in different parts of the world are occupied by different groups of species that happen it have evolved there. If the species naturally had access to everywhere on the globe, we might expect a relatively small number of successful species to become dominant in each biome. The extent to which this homogenisation can happen naturally is restricted by the limited powers of dispersal of most species in the face of the physical barriers that exist to dispersal. By virtue of the transport opportunities offered by humans, these barriers have been breached by an ever-increasing number of exotic species. The effects of introductions have been to convert a hugely diverse range of local community compositions into something much more homogenous.

It would be wrong, however, to conclude that introducing species to a region will inevitably cause a decline in species richness there. For example, there are numerous species of plants, invertebrates and vertebrates found in continental Europe but absent from the British Isles (many because they have so far failed to recolonize after the last glaciations).Their introduction would be likely to augment British biodiversity. The significant detrimental effect noted above arises where aggressive species provide a novel challenge to endemic biotas ill-equipped to deal with them.

Q. With reference to the passage, which of the following statements is correct?

  1. Introduction of exotic species into new geographical areas always leads to reduced biodiversity.
  2. Exotic species introduced by man into new areas have always greatly altered the native ecosystems.
  3. Man is the only reason to convert a hugely diverse range of local community compositions into more homogenous ones.
  4. None of the statements (a), (b), and (c) is correct in this context.

Q. Why does man introduce exotic species into new geographical areas?

  1. To bread exotic species with local varieties.
  2. To increase agricultural productivity.
  3. for beautification and landscaping

Which of the above statements is/are correct?

  1. 1 only
  2. 2 and 3 only
  3. 1 and 3 only
  4. 1, 2 and 3

Q. How is homogenization prevented under natural conditions?

  1. Evolution of groups of species specific to local habitats.
  2. Presence of oceans and mountain ranges
  3. Strong adaptation of groups of specific to local physical and climatic conditions
  4. All the statements (a), (b) and (c) given above are correct in this context.

Q. How have the human beings influenced the biodiversity?

  1. By smuggling live organism
  2. By building highways
  3. By making ecosystems sensitive so that new species are not allowed
  4. By ensuring that new species do not have major impact on local species.

Which of the statements given above are correct?

  1. 1 and 2
  2. 2 and 3
  3. 1 and 3
  4. 2 and 4

Q. What can be the impact of invasion of exotic species on an ecosystem?

  1. Erosion of endemic species.
  2. Change in the species composition of the community of the ecosystem

Select the correct answer using the codes given below:

  1. 1 only
  2. 2 only
  3. both 1 and 2
  4. neither 1 nor 2

Passage-5

Most champions of democracy have been rather reticent in suggesting that democracy would itself promote development and enhancement of social welfare – they have tended to see them as good but distinctly separate and largely independent goals. The detractors of democracy, on the other hand, seemed to have been quite willing to express their diagnosis of what they see as serious tensions between democracy and development. The theorists of the practical spirit - “Make up your mind: do you want democracy, or instead, do you want development?”- often came ,at least to start with , from East Asian countries, and their voice grew in influence as several of these countries were immensely successful – through the 1970 s and 1980s and even later – in promoting economic growth without pursuing democracy.

To deal with these issues we have to pay particular attention to both the content of what can be called development and to the interpretation of democracy (in particular to the respective roles of voting and of public reasoning). The assessment of development cannot be divorced from the lives that people can lead and the real freedom that they enjoy. Development can scarcely be seen merely in terms of enhancement of inanimate objects of convenience, such as a rise in the GNP (or in personal incomes), or industrialisation – important as they may be as means to the real ends. Their value must depend on what they do to the lives and freedom of the people involved, which must be central to the idea of development.

If development is understood in a broader way, with a focus on human lives, then it becomes immediately clear that the relation between development and democracy has to be seen partly in terms of their constitutive connection, rather than only through their external links. Even though the question has often been asked whether political freedom is “conductive to development”, we must not miss the crucial that political liberties and democratic rights are among the “constitutive components” of development does not have to be established indirectly through their contribution to the growth of GNP.

Q. According to the passage, why is a serious tension perceived between democracy and development by the detractors of democracy?

  1. Democracy and development are distinct and separate goals
  2. Economic growth can be promoted successfully without pursuing a democratic system of governance
  3. Non-democratic regimes deliver economic growth faster and far more successfully than democratic ones.
  4. All the statements (a), (b) and (c) given above are correct in this context.

Q. According to the passage, what should be the ultimate assessment/aim/view of development?

  1. Rise in the per capita income and industrial growth rates.
  2. Improvement in the Human Development Index and GNP.
  3. Rise in the savings and consumption trends.
  4. Extent of real freedom that citizens enjoy.

Q. What does a “constitutive” connection between democracy and development imply?

  1. The relation between them has to be seen through external links.
  2. Political and civil rights only can lead to economic development
  3. Political liberties and democratic rights are essential elements of development.
  4. None of the statements (a), (b) and (c) given above is correct in this context.

Passage-6

The need for Competition Law becomes more evident when foreign direct investment (FDI) is liberalised. The impact of FDI is not always pro-competitive. Very often FDI takes the form of a foreign corporation acquiring a domestic enterprise or establishing a joint venture with one. By making such an acquisition the foreign investor may substantially lessen competition and gain a dominant position in the relevant market, thus charging higher prices. Another scenario is where the affiliates of two separate multinational companies (MNCs) have been established in competition with one another in a particular developing economy, following the liberisation of FDI. Subsequently, the parent companies overseas merge. With the affiliates no longer remaining independent, competition in the host country may be artificially inflated. Most of these adverse consequences of mergers and acquisitions by MNCs can be avoided if an effective competition law is in place. Also, an economy that has implemented an effective competition law is in a better position to attract FDI than one that has not. This is not just because most MNCs are expected to be accustomed to the operation of such a law in their home countries and know how to deal with such concerns but also that MNCs expect competition authorities to ensure a level playing field between domestic and foreign firms.

Q. With reference to the passage, consider the following statements:

  1. It is desirable that the impact of Foreign Direct investment should be pro-competitive.
  2. The entry of foreign investors invariably leads to the inflated prices in domestic markets.

Which of the statements given above is/are correct?

  1. 1 only
  2. 2 only
  3. Both 1 and 2
  4. Neither 1 nor 2

Q. According to the passage, how does a foreign investor dominate the relevant domestic market?

  1. Multinational companies get accustomed to domestic laws.
  2. Foreign companies establish joint ventures with domestic companies.
  3. Affiliates in a particular market/sector lose their independence as their parent companies overseas merge.
  4. Foreign companies lower the cost of their products as compared to that of products of domestic companies.

Which of the statements given above are correct?

  1. 1 and 2 only
  2. 2 and 3 only
  3. 1, 2 and 3 only
  4. 1, 2, 3 and 4

Q. What is the inference from this passage?

  1. Foreign investors and multinational companies always dominate domestic market.
  2. It is not in the best interest of domestic economy to allow mergers company.
  3. With competition law, it is easy to ensure a level playing field between domestic and foreign firms.
  4. For countries with open economy Foreign Direct investment is essential for growth.

Passage-7

The poor especially in market economics need the strength that collectivities offer for creating more economic, social and political space for themselves, for enhancing their socioeconomic well-being and voice, and as a protection against free market individualism. It has been argued that a group approach to farming, especially in the form of bottom up agricultural production collectivities, offers substantial scope for poverty alleviation and empowering the poor as well as enhancing agricultural productivity. To realize this potential, however, the groups would need to be voluntary in nature, small in size, participative in decision making and equitable in work sharing and benefit distribution. There are many notable examples of such collectivities to be found in varied contexts, such as in transitions economies. All of them bear witness to the possibility of successful cooperation under given conditions. And although the gender impact of the family cooperatives in the transition economies are uncertain, the Indian examples of women-only groups farming offer considerable potential for benefiting women.

Q. Agricultural collectivities such as group based farming can provide the rural poor

  1. Empowerment
  2. Increased agricultural productivity.
  3. Safeguard against exploitative markets.
  4. Surplus production of agricultural commodities.

Select the correct answer using the codes given below:

  1. 1, 2, 3 and 4
  2. 1, 2 and 3 only
  3. 2 and 4 only
  4. 1, 3 and 4 only

Q. What does the author imply by “gender impact”?

  1. Women are doubtful participants in cooperatives.
  2. Family cooperatives may not include women
  3. Women benefitting from group farming.
  4. Women’s role in transition economies is highly restrictive.

Q. Consider the following assumptions:

  1. It is imperative for transition economies to have agricultural collectivities.
  2. Agricultural productivity can be increased by group approach to farming.

With reference to the above passage which of these assumptions is/are valid?

  1. 1 only
  2. 2 only
  3. Both 1 and 2
  4. Neither 1 nor 2

Passage-8

In a typical Western liberal context, deepening of democracy invariably leads to consolidation of ‘liberal values’. In the Indian context, democratization is translated into greater involvement of people not as ‘individuals’ which is a staple to liberal discourse, but as communities or groups. Individuals are getting involved in the public sphere not as ‘atomized’ individuals but as members of primordial communities drawn on religious or caste identity. Community-identity seems to be the governing force. It is not therefore surprising that the so-called peripheral groups continue to maintain their identities with reference to the social groups {caste, religion or sect} to which they belong while getting involved in the political processes despite the fact that their political goals remain more or less identical. By helping to articulate the political voice of the marginalized, democracy in India has led to ‘a loosening of social strictures’ and empowered the peripherals to be confident of their ability to improve the socio economic conditions in which they are placed. This is a significant political process that had led to a silent revolution through a meaningful transfer of power from the upper caste cities to various subaltern groups within the democratic framework of public governance.

Q. According to the passage, what does “deepening of democracy” mean in the Western context?

  1. Consolidation of group and class identities.
  2. Democratization translated as greater involvement of people.
  3. Democratization as greater involvement of ‘atomized’ individuals in the public sphere.
  4. None of the statements a, b and c given above is correct in this context.

Q. Greater democratization in India has not necessarily led to

  1. The dilution of caste and communal identities in the public sphere.
  2. Irrelevance of community identity as governing force in Indian politics.
  3. Marginalization of elite groups in society.
  4. Relative unimportance of hereditary identities over class identities.

Q. What is the “silent revolution” that has occurred in the Indian democratic process?

  1. Irrelevance of caste and class hierarchies in political processes
  2. Loosening of social strictures in voting behavior and patterns.
  3. Social change through transfer of power from upper caste elites to subaltern groups.
  4. All the statements a), b) and c) given above are correct in this context.

The following eight items are based on three passages in English to test the comprehension of English language. Read each passage and answer the items that follow.

English Language Comprehension

Passage - 1

For fourteen and half months I lived in my little cell or room in Dehradun jail, and I began to feel as if I was almost a part of it. I was familiar with every bit of it, I knew every mark and dent on the whitewashed walls and on the uneven floors and on the ceiling with the moth eaten rafters. In the little yard outside I greeted little tufts of grass and odd bits of stone as old friends. I was not alone in my cell, for several colonies of wasp and hornets lived there, and many lizards found the home behind the rafters, emerging in the evening in the search of prey.

Q. Which of the following explains best the sentence in the passage “I was almost a part of it”?

  1. I was not alone in the cell
  2. I was familiar with every bit of the cell
  3. I greeted little tufts of grass like old friends.
  4. I felt quite at home in the cell

Q. The passage attempts to describe:

  1. The general conditions of the country's jail
  2. The prisoner's capacity to notice the minute details of his surroundings
  3. The prisoner's conscious efforts to overcome the loneliness
  4. The prisoner's ability to live happily with other creatures

Q. The author of the passage seems to suggest that

  1. It is possible to adjust one-self to uncongenial surroundings.
  2. The conditions in Indian prisons are not bad
  3. It is not difficult to spend one's time in prison
  4. There is a need to improve the conditions in our jails.

Passage – 2

We started pitching the highest camp that has been ever made. Everything took five times as long as it would have taken in the place where there was enough air to breathe; but at last we got tent up, and when we crawled in, it was not too bad. There was only a light wind and inside it was not too cold for us to take off our gloves. At night most climbers take off their boots; but I prefer to keep them on. Hilary, on the other hand took his off and lain them next to his sleeping bag.

Q. What does the expression “pitching the highest camp” imply?

  1. They reached the summit of the highest mountain in the world.
  2. Those who climbed that far earlier did not pitch any camp.
  3. So far nobody climbed that high.
  4. They were too many climbers and needed to pitch a big camp

Q. They took a long time to finish the work because:

  1. They were very tired.
  2. There was not enough air to breathe
  3. It was very cold
  4. It was very dark

Q. When they crawled into the tent

  1. They took off their gloves because it was not very cold
  2. They could not take off their gloves because it was very cold.
  3. They took of their gloves though it was very cold.
  4. They did not take off their gloves though it was not very cold.

Passage – 3

A local man, staying on the top of the floor of an old wooden house, was awakened at midnight by fire. Losing his way in the smoke-filled passage, he missed the stairway and went into another room. He picked a bundle to protect his face from fire and immediately fell through the floor below where he managed to escape through a clear doorway. The “bundle” proved to be the baby of the Mayor's wife. The “hero” was congratulated.

Q. The man went to another room because

  1. He did not know where the stairway was
  2. The passage was full of smoke
  3. He was extremely nervous
  4. He stumbled on bundle

Q. The man was called hero because

  1. Expressed his willingness to risk his life to save others
  2. Managed to escape from fire
  3. Showed great courage in fighting the fire.
  4. Saved a life


Civil Services Examination 2011


Read the following passages and answer the questions that follow. Your answers to these items should be based on the passage only.

Passage-1

For achieving inclusive growth there is-a critical need to rethink the-role of the State. The early debate among economists about the size of the Government can be misleading. The need of the hour is to have an enabling Government. India is too large and complex a nation for the State to be able to deliver all that is needed. Asking the Government to produce all the essential goods, create all the necessary jobs, and keep a curb on the prices of all goods is to lead to a large cumbersome bureaucracy and widespread corruption.

The aim must be to stay with the objective of inclusive growth that was laid down by the founding fathers of the nation and also to take a more modern view of what the State can realistically deliver.

This is what leads to the idea of an enabling State, that is, a Government that does not try to directly deliver to the citizens everything that they need. Instead, it (1) creates an enabling ethos for the market so that individual enterprise can flourish and citizens can, for the most part, provide for the needs of one another, and (2) steps in to help those who do not manage to do well for themselves, for there will always be individuals, no matter what the system, who need support and help. Hence we need a Government that, when it comes to the market, sets effective, incentive-compatible rules and remains on the sidelines with minimal interference, and, at the same time, plays an important role in directly helping the poor by ensuring that they get basic education and health services and receive adequate nutrition and food.

Q. According to passage:

  1. The objective of inclusive growth was laid down by the founding fathers of the nation.
  2. Need of the hour is to have an enabling Government.
  3. The Government should engage in maximum interference in market processes.
  4. There is a need to change the size of the Government.

Which of the statements given above are correct?

  1. 1 and 2 only
  2. 2 and 3 only
  3. 1 and 4 only
  4. 1, 2, 3 and 4

Q. According to the passage, the strategy of inclusive growth can be effected by focussing on

  1. meeting all the needs of every citizen in the country.
  2. Increasing the regulations over the manufacturing sector.
  3. Controlling the distribution of manufacturing goods.
  4. Delivery of the basic services to the deprived sections of the society.

Q. What constitutes an enabling Government?

  1. A large bureaucracy.
  2. Implementation of welfare programmes through representatives.
  3. Creating an ethos that helps individual enterprise
  4. Providing resources to those who are underprivileged.
  5. Offering direct help to the poor regarding basic services.

Select the correct answer from the codes given below:

  1. 1, 2 and 3 only
  2. 4 and 5 only
  3. 3, 4 and 5 only
  4. 1, 2, 3, 4 and 5

Q. Why is the State unable to deliver "all that is needed"?

  1. It does not have sufficient bureaucracy.
  2. It does not promote inclusive growth.

Select the correct answer from the codes given below:

  1. 1 only
  2. 2 only
  3. Both 1 and 2
  4. Neither 1 nor 2

Q. What is essential message being conveyed by the author of the passage?

  1. The objectives of inclusive growth laid down by the founding fathers of the nation should be remembered.
  2. The Government needs to make available more schools and health services.
  3. The Government needs to establish markets and industries to meet the needs of the poor strata of the society.
  4. There is a need to rethink the role of the State in achieving inclusive growth.

Passage-2

The concept of 'creative society' refers to a phase of development of a society in which a large number of potential contradictions become articulate and active. This is most evident when oppressed social groups get politically mobilised and demand their rights. The upsurge of the peasants and tribals, the movements for regional autonomy and self-determination, the environmental movements, and the women's movements in the developing countries are signs of emergence of creative society in contemporary times. The forms of social movements and their intensity may vary from country to country and place to place within a country. But the very presence of movements for social transformation in various spheres of a society indicates the emergence of a creative society in a country.

Q. What does the author imply by "creative society"?

  1. A society where diverse art forms and literary writings seek incentive.
  2. A society where social inequalities are accepted as the norm.
  3. A society where a large number of contradictions are recognised.
  4. A society where' the exploited and the oppressed groups grow conscious of their human rights and upliftment.

Select the correct answer using the codes given below:

  1. 1, 2 and 3
  2. 4 only
  3. 3 and 4
  4. 2 and 4

Q. What according to the passage are the manifestations of social movements?

  1. Aggressiveness and being incendiary.
  2. Instigation by external forces.
  3. Quest for social equality and individual freedom.
  4. Urge for granting privileges and self-respect to disparaged sections of the society.

Select the correct answer using the codes given below:

  1. 1 and 3 only
  2. 2 and 4 only
  3. 3 and 4 only
  4. 1, 2, 3 and 4

Q. With reference to the passage consider the following statements:

  1. To be a creative society, it is essential to have a variety of social movements.
  2. To be a creative society, it is imperative to have potential contradictions and conflicts.

Which of the statements given above is/are correct?

  1. 1 only
  2. 2 only
  3. Both 1 and 2
  4. Neither 1 nor 2

Q. Consider the following three statements:

1. Only students can participate in the race.

2. Some participants in the race are girls.

3. All girl participants in the race are invited for coaching.

Which one of the following conclusions can be drawn from the above statements?

  1. All participants in the race are invited for coaching.
  2. All students are invited for coaching.
  3. All participants in the race are students.
  4. None of the statements (a), (b) and (c) given above is correct.

Passage-3

Ecosystems provide people with a variety of goods and services; food, clean water, clean air, flood control, soil stabilization, pollination, climate regulation, spiritual fulfilment and aesthetic enjoyment, to name just a few. Most of these benefits either are irreplaceable or the technology necessary to replace them is prohibitively expensive. For example, potable fresh water can be provided by desalinating sea-water, but only at great cost. The rapidly expanding human population has greatly modified the Earth's ecosystems to meet their increased requirements of some of the goods and services, particularly food, fresh water, timber, fibre and fuel. These modifications have contributed substantially to human well being and economic development. The benefits have not been equally distributed. Some people have actually been harmed by these changes. Moreover, short-term increases in some ecosystem goods and services have come at the cost of the long-term degradation of others. For example, efforts to increase the production of food and fibre have decreased the ability of some ecosystems to provide clean water, regulate flooding and support biodiversity.

Q. With reference to the passage, consider the following statements:

Expanding human' population has an adverse effect on:

  1. Spiritual fulfilment
  2. Aesthetic enjoyment
  3. Potable fresh water
  4. Production of food and fibre
  5. Biodiversity

Which of the statements given above are correct?

  1. 1, 2 and 3 only
  2. 2, 4 and 5 only
  3. 3 and 5 only
  4. 1, 2, 3, 4 and 5

Q. The passage mentions that "some people have actually been harmed by these changes." What does it imply?

  1. The rapid expansion of population has adversely affected some people.
  2. Sufficient efforts have not been made to increase the production of food and fibre.
  3. In the short term some people may be harmed, but in the long term everyone will benefit from modifications In the Earth's ecosystems.

Which of the statements given above is/are correct?

  1. 1 only
  2. 2
  3. 1 and 3
  4. None of the statements given above

Q. With reference to the passage, consider the following statements:

  1. It is imperative to modify the Earth's ecosystems for the well being of mankind.
  2. Technology can never replace all the goods and services provided by ecosystems.

Which of the statements given above is/are correct?

  1. 1 only
  2. 2 only
  3. Both 1 and 2
  4. Neither 1 nor 2

Passage-4

A moral act must be our own act; must spring from our own will. If we act mechanically, there is no moral content in our act. Such action would be moral, if we think it proper to act like a machine and do so. For, in doing so, we use our discrimination. We should bear in mind the distinction between acting mechanically and acting intentionally. It may be moral of a king to pardon a culprit. But the messenger carrying out the order of pardon plays only a mechanical part in the king's moral act. But if the messenger were to carry out the king's order considering it to be his duty, his action would be a moral one. How can a man understand morality who does not use his own intelligence and power of thought, but lets himself be swept along like a log of wood by a current? Sometimes a man defies convention and acts on his own with a view to absolute good.

Q. Which of the following statements best describe/describes the thought of the writer?

  1. A moral act calls for using our discretion.
  2. Man should react to a situation immediately.
  3. Man must do his duty.
  4. Man should be able to defy convention in order to be moral.

Select the correct answer from the codes given below:

  1. 1 only
  2. 1 and 3
  3. 2 and 3
  4. 1 and 4

Q. Which of the following statements is the nearest definition of moral action, according to the writer?

  1. it is a mechanical action based on official orders from superiors.
  2. It is an action based on our sence of discretion.
  3. It is a clever action based on the clarity of purpose.
  4. It is a religious action based on understanding.

Q. The passage contains a statement "lets himself be swept along like a log of wood by a current." Among the following statements, which is/are nearest in meaning to this?

  1. A person does not use his own reason.
  2. He is susceptible to influence/pressure.
  3. He cannot withstand difficulties/challenges.
  4. He is like a log of wood.

Select the correct answer using the codes given below:

  1. 1 only
  2. 1 and 2
  3. 2 and 3
  4. 1 and 4

Passage-5

A country under foreign domination seeks escape from the present in dreams of a vanished age, and finds consolation in visions of past greatness. That is a foolish and dangerous pastime in which many of us indulge. An equally questionable practice for us in India is to imagine that we are still spiritually great though we have come down, in the world in other respects. Spiritual or any other greatness cannot be founded on lack of freedom and opportunity, or on starvation and misery. Many western writers have encouraged that notion that Indians are other-worldly. I suppose the poor and unfortunate in every country become to some extent other-worldly, unless they become revolutionaries, for this world is evidently not meant for them. So also subject peoples.

As a man grows to maturity he is not entirely engrossed in, or satisfied with, the external objective world. He seeks also some inner meaning, some psychological and physical satisfactions. So also with peoples and civilizations as they mature and grow adult. Every civilization and every people exhibit these parallel streams of an external life and an internal life. Where they meet or keep close to each other, there is an equilibrium and stability. When they diverge conflict arises and the crises that torture the mind and spirit.

Q. The passage mentions that "this world is evidently not meant for them". It refers to people who

  1. seek freedom from foreign domination.
  2. live in starvation and misery.
  3. become revolutionaries .

Which of the statements given above is/are correct?

  1. 1 and 2
  2. 2 only
  3. 2 and 3
  4. 3 only

Q. Consider the following assumptions:

  1. A country under foreign domination cannot indulge in spiritual pursuit.
  2. Poverty is an impediment in the spiritual pursuit.
  3. Subject peoples may become other-worldly.

With reference to the passage, which of the above assumptions is/are valid?

  1. 1 and 2
  2. 2 only
  3. 2 and 3
  4. 3 only

Q. The passage thematically centres on

  1. the state of mind of oppressed people
  2. starvation and misery
  3. the growth of civilization
  4. body, mind and spirit of people in general

Q. According to the passage, the torture of the mind and spirit is caused

  1. by the impact of foreign domination.
  2. by the desire to escape from foreign domination and find consolation in visions of past greatness.
  3. due to lack of equilibrium between an external life and an internal life.
  4. due to one's inability to be either revolutionary or other-worldly.

Passage-6

A species that exerts an influence out of proportion to its abundance in an ecosystem is called a keystone species. The keystone species may influence both the species richness of communities and the flow of energy and materials through ecosystems. The sea star Pisaster the flow of energy and materials through ecosystems. The sea star Pisaster ochraceus, which lives in rocky intertidal ecosystems on the Pacific coast of North America, is also an example of a keystone species. Its preferred prey is the mussel Mytilus californianus . In the absence of sea- stars, these mussels crowd out other competitors in a broad belt of the intertidal zone. By consuming mussels, sea star creates bare spaces that are taken over by a variety of other species.

A study at the University of Washington demonstrated the influence of Pisaster on species richness by removing sea stars from selected parts of the intertidal zone repeatedly over a period of five years. Two major changes occured in the areas from which sea stars were removed. First, the lower edge of the mussel bed extended farther down into the intertidal zone, showing that sea stars are able to eliminate mussels completely where they are covered with water most of the time. Second, and more dramatically, 28 species of animals and algae disappeared from the sea star removal zone. Eventually only Mytilus, the dominant competitor, occupied the entire substratum. Through its effect on competitive relationships, predation by Pisaster largely determines which species live in these rocky intertidal ecosystems.

Q. What is the crux of the passage?

  1. Sea star has a preferred prey.
  2. A preferred prey determines the survival of a keystone species.
  3. Keystone species ensures species diversity.
  4. Sea star is the only keystone species on the Pacific coast of North America.

Q. With reference to the passage, consider the following statements:

  1. Mussels-are generally the dominant species in intertidal ecosystems.
  2. The survival of sea stars is generally determined by the abundance of mussels.

Which of the statements given above is /are correct?

  1. 1 only
  2. 2 ony
  3. Both 1 and 2
  4. Neither 1 nor 2

Q. Which of the following is/are implied by the passage?

  1. Mussels are always hard competitors for sea stars.
  2. Sea stars of the Pacific coast have reached the climax of their evolution.
  3. Sea stars constitute an important component in the energy flow in intertidal ecosystem.

Which of the statements given above is/are correct?

  1. 1 and 2
  2. 2 only
  3. 1 and 3
  4. 3 only

Q. Consider the following assumptions:

  1. The food chains/food web in an ecosystem are influenced by keystone species.
  2. The presence of keystone species is a specific characteristic of aquatic ecosystems.
  3. If the keystone species is completely removed from an ecosystem, it will lead to the collapse of the ecosystem.

With reference to the passage, which of the above assumptions is/are valid?

  1. 1 only
  2. 2 and 3 only
  3. 1 and 3 only
  4. 1, 2 and 3

Passage-7

Now India's children have a right to receive at least eight years of education, the gnawing question is whether' it will remain 'on paper' or 'become a reality. One hardly needs a reminder that this right is different from the others enshrined in the Constitution, that the beneficiary - a six year old child cannot demand it, nor can she or he fight a legal battle when the right is denied or violated. In all cases, it is the adult society which must act on behalf of the child. In another peculiarity, where a child's right to education is denied, no compensation offered later can be adequate or relevant. This is so because childhood does not last if a legal battle fought on behalf of a child is eventually won, it may be of little use to the boy or girl because the opportunity missed at school during childhood cannot serve the same purpose later in life. This may be painfully true for girls because our society permits them only a short childhood, if at all. The Right to Education (RTE) has become law at a point in India's history when the ghastly practice of female infanticide has resurfaced in the form of foeticide. This is "symptomatic of a deeper turmoil" in society which compounding the traditional obstacles to the education of girls. "Tenacious prejudice against the intellectual potential of girls runs across our cultural diversity and the system of education has not been able to address it.

Q. With reference to the passage, consider the following statements:

  1. When children are denied education, adult society does not act on behalf of them.
  2. Right to Education as a law cannot be enforced in the country.

Which of the statements given above is/are correct?

  1. 1 only
  2. 2 only
  3. Both 1 and 2
  4. Neither 1 nor 2

Q. According to the passage, what could be the traditional obstacles to the education of girls?

  1. Inability of parents to fight a legal battle when the Right to Education is denied to their children.
  2. The traditional way of thinking about girl's role in society.
  3. The prejudice against the intellectual potential of girls.
  4. Improper system of education.

Select the correct answer from the codes given below:

  1. 1 and 2 only
  2. 2, 3 and 4 only
  3. 1, 3 and 4 only
  4. 1, 2, 3 and 4

Q. On the basis of the passage, consider the following statements:

  1. Right to Education is a legal right and not a fundamental right.
  2. For realising the goal of universal education, the education system in the country must be made identical to that of developed countries.

Which of the statements given above is/are correct?

  1. 1 only
  2. 2 only
  3. Both 1 and 2
  4. Neither 1 nor 2

Q. Which one of the following statements conveys the key message of the passage?

  1. India has declared that education is compulsory for its children.
  2. Adult society is not keen on implementing the Right to Education.
  3. The Right to Education, particularly of a girl child, needs to be safeguarded.
  4. The system of education should be address the issue of right to education.

Q. Which one of the following statements conveys the inference of the passage?

  1. The society has a tenacious prejudice against the intellectual potential of girls.
  2. Adults cannot be relied upon to fight on behalf of children for their Right to Education.
  3. The legal fight to get education for children is often protracted and prohibitive.
  4. There is no sufficient substitute for education received in childhood.

The following nine items are based on three passages in English to test the comprehension of English language. Read each passage and answer the items that follow.


English Language Comprehension

Passage-1

He walked several miles that day but could not get anything to eat or drink except some dry bread and some water, which he got from cottagers and farmers. As night fell, he slept under a haystack lying in a meadow. He felt frightened at first, for the wind blew awfully over the empty fields. He felt cold and hungry, and was feeling more lonely than he had ever felt before. He however, soon fell asleep, being much tired with his long walk. When he got up next day, he was feeling terribly hungry so he purchased a loaf of bread with a few coins that he had.

Q. When the night fell, he slept

  1. in the open field
  2. under a pile of dry grass
  3. in a farmer's cottage
  4. under a tree

Q. He soon fell asleep because

  1. he was exhausted
  2. he was all alone
  3. he had not slept for days
  4. he was very frightened

Q. With reference to the passage, consider the following statements:

  1. He was walking through the countryside,
  2. The cottagers and farmers gave his enough food so that he could sleep at night without feeling hungry.

Which of the statements given above is/are correct?

  1. 1 only
  2. 2 only
  3. Both 1 and 2
  4. Neither 1 nor 2

Passage-2

I opened the bag and packed the boots in; and then, just as I was going to close it, a horrible idea occurred to me - Had I packed my toothbrush? I don't know how it is, but I never do know whether I've packed my toothbrush. My toothbrush is a thing that haunts me when I'm travelling, and makes my life a misery, I dream that haven't packed it, and wake up in a cold perspiration, and get out of bed and hunt for it. And, in the morning, I pack it before I have used it, and it is always the last thing I turn out of the bag; and then repack and forget it, and have to rug upstairs for it at the last moment and carry it to the railway station, wrapped up in my pocket-handkerchief.

Q. When he was going to close the bag, the idea that occurred to him was

  1. unpleasant
  2. sad
  3. fantastic
  4. amusing

Q. What makes his life miserable whenever he undertakes travelling?

  1. Going to railway station
  2. Forgetting the toothbrush
  3. Packing his bag
  4. Bad dreams

Q. His toothbrush is finally

  1. in his bag
  2. in his bed
  3. in his handkerchief
  4. lost

Passage-3

In spring, polar bear mothers emerge from dens with three month old cubs. The mother bear has fasted for as long as eight months but that does not stop the young from demanding full access to her remaining reserves. If there are triplets, the most persistent stands to gain an extra meal and it may have the meal at the expense of others. The smallest of the litter forfeits many meals to stronger siblings. Females are protective of their cubs but tend to ignore family rivalry over food. In 21 years of photographing polar bears, I've only once seen the smallest of triplets survive till autumn.

Q. Female polar bears give birth during

  1. spring
  2. summer
  3. autumn
  4. winter

Q. Mother bear

  1. takes sides over cubs
  2. lets the cubs fend for themselves
  3. feeds only their favourites
  4. see that all cubs get an equal share

Q. With reference to the passage, the following assumptions have been made:

  1. Polar bears fast as long as eight months due to non-availability of prey.
  2. Polar bears always give birth to triplets.

Which of the assumptions given above is/are valid?

  1. 1 only
  2. 2 only
  3. Both 1 and 2
  4. Neither 1 nor 2